QF Book TeX
QF Book TeX
QF Book TeX
Textbook
3 Compound Interest 14
3.1 Compound Interest Formula . . . . . . . . . . . . . . . . . . . . . . . . . . . . . . . . 14
3.2 Simple vs compound interest . . . . . . . . . . . . . . . . . . . . . . . . . . . . . . . 17
3.3 Exercises . . . . . . . . . . . . . . . . . . . . . . . . . . . . . . . . . . . . . . . . . . 18
5 Equation of value 30
5.1 Exercises . . . . . . . . . . . . . . . . . . . . . . . . . . . . . . . . . . . . . . . . . . 33
6 Annuities Certain 36
6.1 Definitions . . . . . . . . . . . . . . . . . . . . . . . . . . . . . . . . . . . . . . . . . . 36
6.2 Ordinary Annuity . . . . . . . . . . . . . . . . . . . . . . . . . . . . . . . . . . . . . 37
6.3 Annuity Due . . . . . . . . . . . . . . . . . . . . . . . . . . . . . . . . . . . . . . . . 39
6.4 Deferred Annuity . . . . . . . . . . . . . . . . . . . . . . . . . . . . . . . . . . . . . . 40
6.5 Perpetuities . . . . . . . . . . . . . . . . . . . . . . . . . . . . . . . . . . . . . . . . . 41
6.6 Common problems with annuities . . . . . . . . . . . . . . . . . . . . . . . . . . . . . 42
6.7 Exercises . . . . . . . . . . . . . . . . . . . . . . . . . . . . . . . . . . . . . . . . . . 44
1
7 Variable Payment Annuities 49
7.1 Introduction . . . . . . . . . . . . . . . . . . . . . . . . . . . . . . . . . . . . . . . . . 49
7.2 Annuities with payments in Geometric Progression . . . . . . . . . . . . . . . . . . . 49
7.3 Annuities with payments in Increasing Arithmetic Progression . . . . . . . . . . . . . 51
7.4 Annuities with payments in Decreasing Arithmetic Progression . . . . . . . . . . . . 53
7.5 Exercises . . . . . . . . . . . . . . . . . . . . . . . . . . . . . . . . . . . . . . . . . . 54
8 Loan Repayment 56
8.1 Preliminaries . . . . . . . . . . . . . . . . . . . . . . . . . . . . . . . . . . . . . . . . 56
8.2 Amortization Schedules . . . . . . . . . . . . . . . . . . . . . . . . . . . . . . . . . . 56
8.2.1 Constant Principal Loan . . . . . . . . . . . . . . . . . . . . . . . . . . . . . . 56
8.2.2 Level Payment Loan . . . . . . . . . . . . . . . . . . . . . . . . . . . . . . . . 57
8.3 Common problems with amortization schedules . . . . . . . . . . . . . . . . . . . . . 59
8.4 Exercises . . . . . . . . . . . . . . . . . . . . . . . . . . . . . . . . . . . . . . . . . . 61
9 Leasing 66
9.1 Exercises . . . . . . . . . . . . . . . . . . . . . . . . . . . . . . . . . . . . . . . . . . 68
10 Bonds 73
10.1 Definitions . . . . . . . . . . . . . . . . . . . . . . . . . . . . . . . . . . . . . . . . . . 73
10.2 Amortization Schedules . . . . . . . . . . . . . . . . . . . . . . . . . . . . . . . . . . 75
10.3 Bond Valuation . . . . . . . . . . . . . . . . . . . . . . . . . . . . . . . . . . . . . . . 76
10.4 Exercises . . . . . . . . . . . . . . . . . . . . . . . . . . . . . . . . . . . . . . . . . . 79
11 Share Valuation 84
11.1 Definitions . . . . . . . . . . . . . . . . . . . . . . . . . . . . . . . . . . . . . . . . . . 84
11.2 Valuation . . . . . . . . . . . . . . . . . . . . . . . . . . . . . . . . . . . . . . . . . . 85
11.3 Exercises . . . . . . . . . . . . . . . . . . . . . . . . . . . . . . . . . . . . . . . . . . 87
2
List of Figures
3
Chapter 1
People and organizations in general can use their available income in two main ways: Either in
consumption or in savings. Consumption can be defined as the expenditure in goods or services,
having usually a defined lifetime, does not allow any return, apart from the satisfaction of the
item consumed. Saving can be transformed into liquid currency without any type of income or
into investment. So, we can roughly divide people into consumers and investors. In this course
we deal basically with investments and investors, and we deal quantitatively some basic financial
operations, quantitatively only.
Definition 1.1 (Financial Operation) It is some operation that intends to change quantitatively
some amount of money, called Principal, and is done for some time interval.
The financial operation always is referred to a time period, an interval of time, with a beginning
and an end (it can be infinity).
Definition 1.3 (Principal) A Principal, or capital, is some amount of money or capital that is
operated financially, invested or borrowed, generating more capital.
A principal, always refers to an instant in time, when is borrowed, or lent, it produces interest. Its
application is done for a defined period of time, and the interest produced can be put available at
any instant of some time interval, commonly at the end, sometimes at the start.
Definition 1.4 (Interest) Interest is the cost for the use of borrowed money. It is a function of
the capital applied, as well as its term.
The interest is the price of money use, it is given through a rate related to a given period of time.
Definition 1.5 (Interest rate) The interest rate is given through a percentage relative to a unit
of capital and always refers to a given time period.
4
Definition 1.6 (Accumulation Period) It is the interest due time period, when the interest is
calculated, then either paid or reinvested.
Most often it is given referring to a period of time of one year, but it can be referred to any period
of time other than the year, often some division of the year. As an example, we can have an annual
interest rate, but also semi-annual, quarterly, monthly, weekly, even daily. In some particular
operations we can even refer interest to be charged in continuous time, in that case we don’t have
an interest rate but an Interest Force.
Example 1.1 An Accumulation Period can be annual, semi-annual, quarterly, etc. Subsequent
Payment Periods can be similar, as well as interest rates. We can have rates such as (often we
add a subscript to refer the time period): iA = 5%: 5% per annum, per year, iS = 2.5%: 2.5% per
semester, semi-annual rate, iM = 1.5%: 1.5% per month.
Remark: They do not need to be proportional. Often they aren’t, we’ll deal with this later.
For simplicity, we consider that each month has 30 days and the year has 360 days. This follows
what in the U.S. is called as the Banker’s Rule, where the year has 360 days and any fractional
part of the year is the exact number of days of a loan.
We introduce now the basic interest formula for a time period, interest is given multiplying the
principal with the interest rate. The interest rate must be referred to the Accumulation Period
(or equivalent). The interest plus the principal gives the accumulated capital or the Accumulated
Value or Future Value.
Formula 1.1 (Basic Interest Formula) Let I be the interest, P the principal, i the interest
rate referred to the accumulation period, the period of the operation. Then the interest for the given
period is obtained by
I = P × i. (1.1)
Once calculated the interest after some period of time we can get what is called as the Accumulation
Value. See Figure 1.1.
Formula 1.2 (Accumulation Value/Future Value, Present Value) Let P be the Principal,
I be the interest and F V the Future Value or the Accumulation Value after some time period. The
Accumulation Value is given by
FV = P + I (1.2)
and the Present Value is given by the backwards operation
PV = FV − I . (1.3)
In the formula, the Principal P corresponds to the Present Value of a Capital: P = P V . This
operation result (P V ) can also be considered as a Discount Value, although we have to take care
because the word Discount can have different meanings according to diverse concepts.
Interest calculation can be different according to each concept. If we considered discrete time,
time is composed by countable periods, we can summarize interest calculation in two different
regimes: The Simple Interest Regime and Compound Interest Regime (A mixed regime could also
5
P FV = P + I
0 t
P rincipal Accumulation
be considered, however uncommon). Apart from this we could consider accumulation computed
continuously in time, it is mathematically more complex and has its applications in more elaborated
financial operations. For now it is out of scope.
In the next two chapters we are going to deal with Simple Interest and Compoud interest, where
accumulation is done in discrete time. In Simple Interest Regime the interest produced in each
period is paid, whereas in Compound Interest Regime the interest produced is retained for later
payment.
1.2 Exercises
1. True/False questions:
(a) For a given non-negative interest rate, the accumulated value of a principal at the end
of the application period is always greater than the principal itself.
T F
(b) Mr. Zeus did an application of e 1, 000 at an effective interest rate of 10%. This means
that at the end of the accumulation period his principal grew 10% higher.
T F
(c) A quarterly effective interest rate means that an application is made for 4 months.
T F
(d) Mrs. Zeus did this year an application of e 1, 000 at a quarterly interest rate of 4%. This
can mean that the application pays interest four times in a year.
T F
(a) Relative to the above Mr. Zeus’ e 1, 000 application at a quarterly interest rate of 10%,
it results on an interest of
6
4. Mrs. Zeus invested a Principal at a quarterly interest rate of 4% and obtained e 1, 000 at the
end of the application period. Calculate the Principal.
7
Chapter 2
Simple Interest Regime is characterized by the fact that interest produced in each accumulation
period is paid immediately, we mean, gets out of the process.
We exemplify with a picture. First, for simplification assume that Interest gains are constant
throughout time and the accumulation periods have equal length, i.e., interest in each period only
varies if the interest rate varies). As it is shown in Figure 2.1, the Interest produced in each time
period is not added to the Principal, where P is the initial capital application during t time units
and interest generated for each period k is Ik .
% % % %
I1 I2 I3 It
P → P → P → P −→ ··· −→ P
0 1 2 3 t (tempo)
Formula 2.1 (Simple Interest calculation) The interest produced throughout the term of the
operation is given as: Let I be the interest per time unit, P the Principal, t the Time (in time
units), i Interest Rate, referred to the time unit:
I = P × i × t = P it, t ≥ 0. (2.1)
Formula 2.2 (Accumulation Value or Future Value in Simple Interest) The Accumulation
Value or Future Value (F V ) is given by formula:
F V = P + I = P (1 + i t) , t ≥ 0. (2.2)
Remark 2.1 Although Formulae (2.1) and (2.2) were illustrated in Figure 2.1 as if time were
discrete (t = 0, 1, 2, . . . ) they are indeed valid, extended, for a real non-negative t ≥ 0. Also, we
8
note that the rate i is referred to a time unit t = 1, without loss of generality we can interpret it as
an annual rate, where time is measured in years.
Formula 2.3 (Present Value) Equivalently, solving Equation (2.2) with respect to P V , Present
Value is given by
FV
PV = P = = F V (1 + i t)−1 . (2.3)
(1 + i t)
For a unit period of time t = 1, for one year, we have the formulae above simplified to the following.
Formula 2.4 (Future Value for t = 1) The Accumulation Value or Future Value (F V ) is given
by formula:
F V = P (1 + i) . (2.4)
Formula 2.5 (Present Value for t = 1) Equivalently, solving Equation (2.2) with respect to P V ,
Present Value is given by
P V = P = F V (1 + i)−1 . (2.5)
From Formula (2.4), (1 + i) is called the Accumulation Factor, and from Formula (2.5), v =
(1 + i)−1 is known as the Discount Factor.
Remark 2.2 We see that at the end of the Term the total interest payment is I = P i t, equivalently
we can write I = it , where it = i t is the interest rate referred to the period t. This means that the
two interest rates are proportional, and they are equivalent in the sense that they produce the same
accumulated value for the same term. There is a correspondence between the two rates and that
correspondence is proportional.
Example 2.1 In the Term, Time Periods can be: Annual, Semi-annual, Quarterly, Monthly,
etc. Corresponding to the Year, the Semester, the Quarter, the Month, etc., respectively.
Similarly, the Interest Payment Periods can be: Annually: once a year; Semi-annually: Twice
a year; Quarterly: 4 times a year; Monthly etc., so that Interest Rates are Annual, Semi-annual,
Quarterly, Monthly and so on.
Example 2.2 Consider that Mr. Zed puts e 10, 000 in a savings account for a term of one year,
paying 3% simple interest on every quarter. He wants the interest to finance a regular expense that
is due every quarter for one year. Calculate the interest earned in every quarter, as well as the
accumulated value after one year.
Solution: The quarterly interest is equal on every 3-month period and is equal to e10, 000(0.03) =
e300. The Future Value, or Accumulated Value, is
Note that the 3% quartely interest rate is equivalent to an annual interest rate of 12%.
9
Simple Interest is of limited interest, it is mainly applied in Short Term financial operations, like
in small loans for short time. Short Term here means terms less than a year.
We can generalise the simple interest accumulation formula (2.2) considering the case where the
interest does not necessarily remains constant on every accumulation period.
Formula 2.6 (Future Value with variable Interest rate) It’s easily understandable that the
Future Value (F V ) for a term of t = 1, 2, . . . years (or equal periods) is given by formula:
t
!
X
FV = P 1 + ik , (2.6)
k=1
In some short term operations, like short term bank loans also known as Bank Discount, it is
demanded that interest is paid upfront. This corresponds to inverting the normal financial process
where the compensation for providing the service of a loan is done since the service is completed.
Paying interest upfront disguises the real cost of a loan, its reversed business like what happens in
insurance, where the customer pays beforehand.
Definition 2.1 (Simple Discount or Bank Discount) Simple Discount Interest or simply Sim-
ple Discount, or even Bank Discount, is the interest whose calculation is based on the Future Value
rather than on the Present Value, its payment is due at the beginning of the operation term.
The money that is borrowed is called the Amount corresponding to the Future Value. The interest
charged for the use of borrowed money is called the Simple Discount or simply Discount. This is
easily understood since the charge is paid upfront the amount received from the loan comes already
discounted of interest. The rate that is used to compute the interest charges is called the Discount
Rate. The money received is called Proceeds or Principal and it is the Present Value. A diagram
is shown in Figure 2.2, and in Formula 2.7 we see how the calculations are done.
P FV
0 t
P rincipal Amount
Formula 2.7 (Simple Discount Formula) Let P be the Principal, F V the Amount or Future
Value, D the Discount, d the Discount Rate and [0, t] the accumulation period. Then
D = FV × d × t
P = FV − D ;
10
Equivalently,
P = F V (1 − d t)
P
FV = .
(1 − dt)
• Promissory Note: Document on which one party writes his or her promise to pay another
party the principal and the interest for a loan due at some date in the future.
• Treasury Bills, or T-Bills: Short-term loans to the U.S. federal government, carrying terms
ranging from a few days to 6 months, though the most common terms are 4, 13, or 26 weeks
• Commercial Paper : A type of promissory note used by corporations, and is short-term.
Example 2.3 (Discount) A e 10, 000 face value discount note has a term of four months. The
discount rate is 6% per year. Find the amount of the discount.
Solution:
D = 10, 000 × 0.06 × (4/12) = 10, 000 × 0.02 = 200
Note that the application of an interest rate of 6% per year for four months is equivalent to the
application of an interest rate of 2% per four months, the interest rates are proportional to time of
the application.
Note that the Discount Rate is calculated over the Future Value and paid upfront wheras normal
Interest is paid over the Principal and paid at the end. In the following example we underline the
differences in the concepts, and that lead to different values.
Example 2.4 Consider that Mr. Zed borrows e1, 000 for a time period of one year, paying 10%
interest upfront. That is being discounted to the Amount: P = 1, 000(0.1) = 900, so that Mr. Zed
is charged a Discount of e100. At the end of the year he is paying back F V = e1, 000. The annual
Interest Rate, iA , Mr. Zed is paying effectively is:
100 1000 − 900 1
iA = = = = 11.1(1)% 6= 10% .
900 900 9
This is different from the 10% Mr. Zed was said to be paying for interest. Indeed, he is being charged
through a Discount Rate rather that an Interest Rate. Discount Rate dA = 10% and Interest Rate
iA = 11.1(1)%, so that:
900(1 + 0.111(1)) = e1, 000; Discount = e100 .
We can easily establish a general relationship between the interest rate and the discount rate, in
simple interest regime. This is done in Result 1 below.
Result 1 (Interest versus discount rate) In simple interest regime, interest rate i and discount
rate d, referred to the same time period, are related through formulae
i d
d= ⇔i= .
1+i 1−d
11
Proof. Discount Rate d is given by:
Discount FV − PV
d= = ⇔ P V = F V (1 − d) .
F uture V alue FV
On the other hand, present value P V using interest rate calculation in (2.5) we get
i d
F V (1 − d) = F V (1 + i)−1 ⇔ d = ⇔i= .
1+i 1−d
2.3 Exercises
1. True/False questions:
(a) For a fixed interest rate, in Simple Interest the interest value in each period is always
constant.
T F
(b) Simple interest accumulation is mostly used in short term operations.
T F
(c) Consider simple interest. For the fixed interest rate iA =10%, an application takes ten
years to duplicate its (initial) principal.
T F
(d) Discounting a principal is a financial operation known as simple discount.
T F
(e) Simple discount corresponds to the present value of a unit principal discounted one year.
T F
(f) For a positive rate and same period, in Simple Interest discount rate and interest rate
are the same.
T F
(g) Simple Discount and Simple Interest are different concepts that can be mathematically
related, equated.
T F
(h) Let Dr. Zen’s statement: Simple discount simply means that interest, over the money
borrowed, is paid in advance.
T F
(a) Consider simple interest and an annual rate of 6%. At he end of three years Mr. Zed
got an accumulated value of e 2 950, 00. Approximately, what is the Principle applied?
12
1. e 2 783, 02 × ; 2. e 2 419, 00 × ; 3. e 2 500, 00 × ; 4. Outro × .
(b) Consider simple interest and a semi-annual interest rate of 2.5%. Calculate the necessary
investment amount that Dr. Zen needs to do in order to accumulate a total of e1, 050.00
within 24 months:
1. e 1, 001.14 × ; 2. e 1, 000.00 × ; 3. e 954.54 × ; 4. Other × .
(c) Consider simple interest and a monthly interest rate of 1.2%. Dr. Zen got a loan
of e57, 000.00. Principal and interest were paid altogether once at the end summing
e63, 839.54. What is the loan term? Approximately:
1. 1 year, 2 months and 3 days × ; 2. 9 months and 10 days × ;
3. 9 months and 15 days × ; 4. Other × .
(d) In simple interest, Mr. Zen did a four year application of e 10, 000 at a nominal (annual)
rate of 2% with semi-annual accumulation. Write the interest earned in the third year,
approximately.
1. e 300.00 × ; 2. e 200.00 × ; 3. e 209.16 × ; 4. Other × .
(e) Consider simple interest and monthly interest rate of 0.5%. Dr. Zen has to pay 300, 600
and 900 euros within 3, 6 and 12 months, respectively. After six months from start he
has not made any payment. At that time what is Dr. Zens complete debt?
1. e 1, 778.29 × ; 2. e 1, 800.00 × ; 3. e 1, 752.62 × ; 4. Other × .
3. Assuming simple interest and the following options for a Dr. Zen’s 6 month application,
indicate and explain by calculation the correct sequence of income terms:
(a) Interest rate iT = 5% for one quarter, following by an interest rate iS = 10% for the
other quarter;
(b) Interest rate iT = 3% for two quarters, and an interest rate iM = 4% for the following
months;
(c) Interest rate iM = 3% for three months, and iQ = 4% for one quarter.
(d) Interest rate iT = 4% for one quarter, and i2M = 3% for three months.
13
Chapter 3
Compound Interest
Assume now for simplification that the Interest Rate, i, for each accumulation year or period is
constant, and every accumulation period have the same time length, for the whole of the financial
operation. In Compound Interest Regime, Interest produced in each year is added to the Principal,
so that at the begining of each year the Principal is varying, increasing if the interest rate is positive.
We get a sequence, index refers to the ordered year, for the term [0, t]: {P0 , P1 , · · · Pt−1 , Pt }. This
dynamics is exemplified below in Figure 3.1, together with the equations in (3.1).
I1 I2 I3 It
↓ ↓ ↓ ↓
P0 → P1 → P2 → P2 −→ ··· −→ Pt
0 1 2 3 t (time)
P1 = P0 + I1 = P0 + P0 · i = P0 (1 + i)
P2 = P1 + I2 = P0 + I1 + I2 = P1 (1 + i) = P0 (1 + i)2
X3
P3 = P2 + I3 = P0 + Ik = P2 (1 + i) = P0 (1 + i)3
k=1
...
t
X
Pt = Pt−1 + In = P0 + Ik = Pt−1 (1 + i) = P0 (1 + i)t , t = 0, 1, 2, .... (3.1)
k=1
14
Formula 3.1 (Future Value or Accumulated Value Formula) Let P = P V and S = F V ,
FV = S = P (1 + iA )t , t ≥ 0 , and, (3.2)
Xt
= P+ Ik , for t = 1, 2, . . .
k=1
where (1 + iA ) is the Annual Accumulation Factor, interest rate i is referred to time unit of t, and
Ik is the interest generated in year k.
From Formula (3.2) we can calculate the Future Value Formula, solving equation with respect to
P , this is in the formula below.
S
P = S(1 + iA )−t = = S vt , t ≥ 0 ; (3.3)
(1 + iA )
t
X
P = FV − Ik , for t = 1, 2, . . .
k=1
where v = (1 + iA )−1 is the Annual Discount Factor, interest rate i is referred to time unit of t,
and Ik is the interest produced in time unit k.
Example 3.1 In Compound Interest, using Formula (3.2) in the first quarter Mr. Zed accumulates
the same as in Simple Interest:
whereas in Simple Interest we had e11, 200.00. After four quarters we have a difference of e55.09
plus for the compound regime.
From now on we present our final values always rounded at the e cent. Also, we write the year as
it could be any period of time, we mean for instance that if there were no mistake or confusion,
15
when we talk on “annual periods” or “annual rates” it may mean referring to any given period,
without loss of generality.
The interest generated in each period of time, say k it is simply given by the product Ik = Pk−1 ×iA .
That is simple to be calculated, however the calculation of all the interest generated after t years
is more complex. Following formulae show these calculations
Formula 3.3 (Interest generated after t years) In Compound Regime, the interest generated
I after t years is given by
t
X
Ik = F V − P0 = Pt − P0 = P0 (1 + iA )t − P0 = P0 (1 + iA )t − 1 ,
I=
k=1
Ik = Pk−1 × i = P0 (1 + i)k−1 × i
Formulae for computation of Future Value, and subsequently to Present Value, can be generalised
easily when the interest varies from year to year: ik , k = 1, . . . , t:
In the next section we study the behaviour of the two accumulation systems.
16
3.2 Simple vs compound interest
In the Simple Interest Accumulation Formula we have that the Future Value at time t, with t ≥ 0
(s) (s)
and denoted as St , is given by: St = P (1 + i t), where i is an annual interest rate. Likewise, in
the Compound Interest Accumulation Formula, we have that the Future Value at time t, denoted
(c) (c)
as St is given by St = P (1 + i)t , and we consider the same interest rate as well as the same
time domain t ≥ 0 (superscripts (s) and (c) refer to “simple” and “compound”, respectively). First,
consider the example that follows.
Example 3.2 Mr. Zen puts the same e100, 000 amount in two applications, one in Simple Interest
and the other one in Compound one, where the same interest rate is considered: iA = 30%. In
Table 3.1 we have figures of Accumulated values in both interest regimes for several time instants,
the last column shows the difference ‘Compound minus ‘Simple’. We see that the difference is
negative for time 0 < t < 1 and is positive for t > 1, it is zero at t = 0, 1 and it clearly boosts
positively as time is higher and higher. We remark that when t = 1 the two functions St give the
same value, the “unit” corresponds to the period of the interest rate. If the interest rate is annual,
the unit is one year, if we have a quarterly rate then the time unit is the quarter, and similarly to
other reference time periods.
(s) (c) (c) (s)
t St St St − St
0.01 100,300.00 100,262.71 -37.29
0.03 100,900.00 100,790.20 -109.80
0.05 101,500.00 101,320.46 -179.54
0.1 103,000.00 102,658.36 -341.64
0.3 109,000.00 108,188.97 -811.03
0.5 115,000.00 114,017.54 -982.46
0.7 121,000.00 120,160.12 -839.88
0.9 127,000.00 126,633.62 -366.38
1 130,000.00 130,000.00 0.00
2 160,000.00 169,000.00 9,000.00
3 190,000.00 219,700.00 29,700.00
4 220,000.00 285,610.00 65,610.00
5 250,000.00 371,293.00 121,293.00
6 280,000.00 482,680.90 202,680.90
8 340,000.00 815,730.72 475,730.72
10 400,000.00 1,378,584.92 978,584.92
Figure 3.2 shows a typical behaviour of the two accumulation processes along time, where two
intersection points exist only, at “0” and at “1”. It is understandable why simple interest is
preferred for short term bank loans and compound interest for other terms, it is when banks charge
more. In order to maximize interest a Mixed Regime could be used, Compound for integer annual
periods and Simple for fractions of a period. See Example 3.3.
17
Figure 3.2: Accumulation curves for Simple and Compound Interest
Example 3.3 Consider that Mr. Zen did an application of e10, 000 for four years and five months
in a mixed accumulation system, at a 10% annual rate. The Future Value will be:
F V = P4+5/12 = 10, 000(1.1)4 [1 + (5/12)(0.1)] ' 15, 251.04 .
For the same period, with Simple Interest we would get F V (s) = 14, 416.67 and for Compound
F V (c) = 115, 234.13 (superscript (s) or (c) identifies the interest regime).
One more, an important remark. We’ve presented the Simple and Compound Interest Regimes,
and we’ve remarked that Simple Interest is applied in short term operations, basically in banking
short term operations. It has indeed a quite limited application, although it is easy to operate by
common people as everything is proportional. The regime that really matters is the Compound
one, as it fits better the real financial movements, as far as discrete time is concerned. From this
point on, this manuscript deals mostly with Compound Interest, except where explicitly mentioned.
3.3 Exercises
1. True/False questions:
(a) For a positive interest rate, the accumulated value of a principal in simple and compound
regimes is always different, no matter the application time.
T F
(b) Consider a positive annual interest rate, and the statement: Similarly to a stopped clock
and the right hour there are only two moments in time that the accumulation value of a
given principal in compound interest and simple interest are the same.
T F
(c) For an interest rate i = 10%, a principal applied under simple interest takes about
+37, 5% more time to double the accumulated capital than the same application if done
under compound interest.
T F
2. Multiple choice-single answer questions:
(a) Consider compound interest. Mr. Zen did a 4-year application of amount e 10 000 at a
semi-annual rate of 1%. The accumulated value at end of term is, approximately,
18
1. e 10, 800.00 × ; 2. e 10, 406.04 × ; 3. e 10, 828.57 × ; 4. Other × .
(b) Consider compound interest. Mr. Zeus did a 2-year application which accumulated
an amount of e 10 828.57 at a quarterly rate of 1%. The initial capital, Principal, is
approximately,
1. e 10, 026.45 × ; 2. e 10, 000.00 × ; 3. e 10, 615, 20 × ; 4. Other × .
(c) Consider compound interest and a quarterly rate of 1%. Dr. Zuca did an application of
e 10 000, 00 having accumulated at maturity an amount of e 10, 406.20. Calculate the
term of the operation, approximately.
1. One year × ; 2. Four years × ; 3. Two years × ; 4. Other × .
(d) In compound interest, Mr. Zen did a four year application of e 10, 000 at a nominal
(annual) rate of 2% compounded semi-annually. Write the interest earned in the third
year, approximately.
1. e 105.10 × ; 2. e 102.01 × ; 3. e 209.16 × ; 4. Other × .
(e) Consider compound interest and monthly rate of 0.5%. Dr. Zen has to pay 300, 600 and
900 euros within 3, 6 and 12 months, respectively. After six months from start he has
not made any payment. At that time, what is Dr. Zens complete debt?
1. e 904.52 × ; 2. e 873.47 × ; 3. e 1, 777.99 × ; 4. Other × .
(f) Zach SA signed today an acquisition contract for an industrial equipment with a payment
plan with two instalments. Values and due dates are: from today, e 6, 400 within six
months and e 7, 200 within 18 months, respectively. Compound interest and an annual
interest rate of 6% are applied, compute the acquisition value of the equipment.
1. e 12, 793.10 × ; 2. e 12, 813.64 × ;
3. e 13, 600.00 × ; 4. Other × .
(g) Mr. Brad invested e 2, 000 for two years at an annual rate of 5% and after during a
certain period at an yearly rate of 4%, having accumulated e 2, 432.16 in the end of
term. For compound interest, what was the complete application period?
1. 4 years × ; 2. 4.5 years × ;
3. 5 years × ; 4. Other × .
(h) Consider Mr. Zeds incomplete statement: “For a given positive interest rate, it is better
to make an application of an amount of money in compound interest then in simple
interest, ” . Complete with the best correct option:
1. if the application period is less than a year. × ;
2. depending on the interest rate size. × ;
3. if the application period is greater than a year. × ;
4. depending on the reference period of the interest rate. × .
3. Assuming compound interest and the following options for a Dr. Zen’s 6-month application,
indicate and explain by calculation the correct sequence of income terms:
(a) Interest rate iT = 5% for one quarter, following by an interest rate iS = 10% for the
other quarter;
19
(b) Interest rate iT = 3% for two quarters, and an interest rate iM = 4% for the following
months;
(c) Interest rate iM = 3% for three months, and iQ = 4% for one quarter.
(d) Interest rate iT = 4% for one quarter, and i2M = 3% for three months.
4. Using Formulae 3.5 show that the total interest produced by an investment P0 at the end of
t years can be obtained through equation
t
" t #
X Y
Ik = P0 (1 + ik ) − 1 , t = 0, 1, 2, . . . ,
k=1 k=1
where Ik and ik are the interest produced in year k and respective interest rate.
20
Chapter 4
There are several concepts of interest rates as well as other sorts of rates that sometimes people
mistake with interest rates. We must note that interest is conceptually the compensation for the
use of money, is a service payment for using other people’s money, or the (relative) cost of money.
It must not be confused with return rate, for instance, that is the (relative) compensation for an
investment, or a cost rate which carries all costs of some financial activity, often including interest.
Let us first talk about Interest Rates. Amongst the different concepts we name the following. Most
often, banks and other institutions, present to clients and investors the Nominal Interest Rates.
These, are rates proportional to the Effective ones.
Definition 4.1 (Annual Nominal Rate with m Conversion Periods) Let i(m) denote the Nom-
inal Interest Rate. It is the annual percentage rate, that is usually presented to clients, where m is
the (integer) number of Conversion Periods per year, m = 1, 2, . . . , and a Conversion Period is the
time between two successive interest generations, or interest accumulation.
Definition 4.2 (Effective Rate) Let i be the effective rate. It is the interest rate per conversion
period, proportional to i(m) :
i = i(m) /m .
Although the Nominal Rate is defined as an annual rate, annual is taken here as a general fixed
period of time. It could be referred to a semester, quarter or any other time unit, previously defined.
Also, although we could consider m = 1, in practice it is unusual because in that way the nominal
rate and the effective rate would coincide.
Example 4.1 Ms. Zelda made an one year application of P = e1, 000.00 at rate of 10% converted
semi-annually. The annual Nominal Rate is: i(2) = 10% and Effective rate: i(2) /2 = 5% .
The Nominal and the Effective rates are proportional, we can in general talk about proportional
rates. The year period is two times the semester one and the two rates have the same proportionality.
Definition 4.3 Two rates are proportional when there is a proportional relationship between the
rates according to the periods they refer to.
21
However, two proportional rates may not be equivalent as it may be dependent on the interest
regime. First, we give the concept.
Definition 4.4 (Equivalent Rates) For the same term, two rates are said to be equivalent if
they produce the same accumulated, or present, value.
Example 4.2 Let us recover Example 4.1, we have that nominal rate is 10%, and effective rate is
5%.
With simple interest calculation, after one year
F V = 1, 000(1 + 2(0.5)) = 1, 000(1 + 0.1) = 1100 ,
then the rates iS = 5% and iA = 10% that are proportional are also equivalent. This is true in
general, see Remark 2.2.
In the case of compound interest, this is a different story. Let
1, 000(1.05)2 = 1, 102.50 and
1, 000(1.1) = 1, 100.00 .
Rates 5% and 10% are not equivalent, we can use this as a counterexample to say that in compound
interest proportional rates are not equivalent, in general. Indeed, we see that rate i∗A = 10.25%
1, 000(1.1025) = 1, 000(1.05)2 = 1, 102.50
is equivalent to iS = 5%. We could use the equation to find the annual equivalent rate
(1 + i∗A ) = (1.05)2 .
Let’s consider now only Compound Interest and define “APY”, meaning Annual Percentage Yield.
APY is also known as the Annual Equivalent Rate (briefly, AER) or as Effective Annual Rate
(EAR).
Proof.
!m !
i(m) 1 i(m)
(1 + i) = 1+ ⇔ (1 + i) m = 1+
m m
h 1
i
i(m) = m (1 + i) m − 1 .
22
Formula 4.2 (Nominal Rate calculation)
!m
i(m)
(1 + i) = 1+
m
!
1 i(m)
(1 + i) m = 1+
m
h 1
i
i(m) = m (1 + i) m − 1
In Compound Interest, we can relate Annual Nominal interest rates with different conversion peri-
ods, see the Formula that follows.
Formula 4.3 (General Equation for Equivalent Rates) Consider compound interest and an-
nual Nominal Interest Rates i(m) and i(m) , with m, n = 1, 2, . . . , and Annual Equivalent Rate iA ,
then
!m !n
i(m) i(n)
1+ = 1+ ,
m n
and
!n/m
i(n)
i(m) = m 1+ − 1
n
!m/n
i(m)
i(n) = n 1+ − 1
m
Proof.
(1 + i(m) /m)m = 1 + iA
⇒ (1 + i(m) /m)m = (1 + i(n) /n)n , and
(1 + i(n) /n)n = 1 + iA
1 + i(m) /m = (1 + i(n) /n)n/m ⇔ (1 + i(m) /m)m/n = 1 + i(n) /n ,
or
i(m)
= (1 + i(n) /n)n/m − 1
m
i(n)
= (1 + i(m) /m)m/n − 1 .
n
There are other sorts of rates, that aren’t necessarily interest rates, instead are reference rates
as basis to calculate bank interest rates, for instance. One example are the Euribor Rates (see
euribor-rates.eu)
23
Definition 4.6 (Euribor Rates) Euribor is short for Euro Interbank Offered Rate. The Euribor
rates are based on the average interest rates at which a large panel of European banks borrow funds
from one another.
The Euribor rates are considered to be the most important reference rates in the European money
market. The interest rates do provide the basis for the price and interest rates of all kinds of
financial products.
There are sorts of other rates, although commonly known as interest rates, are cost rates, that
include interest cost amongst other costs, like admin costs and taxes. In the Portuguese market some
of these kind of rates are even regulated by law and target consumer credit, usually with shorter
terms. This is to provide that the consumer knows the real cost of the credit on a rate. Usually,
banks and other financial operators, provide at the top a nominal Rate, called the “TANB”, and
they have to disclose the rate that include all costs:“TAEG” and “TAE”. For details see (Barroso
et al., 2009, pp 259-267).
Definition 4.7 (TANB: Gross annual nominal rate) It is an annual nominal rate that the
operators give to clients, it is the basis to calculate the effective interest rate according to the term
of the operation.
Definition 4.8 (TAEG: Effective annual global rate) It is an annual rate that includes all
costs associated to the credit, including interest as well as taxes. It is then applied to the credit
term under Compound Interest.
Definition 4.9 (TAE: Effective annual rate) It is similar to the TAEG rate excluding the in-
terest tax. Since the tax is paid by the customer, it is not a cost for the banker.
We give an example, taken from Barroso et al. (2009), of how to calculate these rates.
Example 4.3 (TAEG and TAE calculation) Mr. Zappa got a consumer credit of amount e4, 500.00
with a five month term, to be paid back as a lump sum at maturity. Costs are summarized as: In-
terest (I) at annual nominal rate of 9.2%, calculated under Simple Interest; Admin costs (A) of
e45.00e; e18.25 as Management costs (M ); e8.45 as Repayment cost R; Tax (T ) at 4%.
Calculation of the Amount, F V , where t is the maturity:
FV = P0 + (I + T ) + A + M + R
9
= 4, 500 + 4, 500 × × 0.092(1 + 0.04) + 45 + 18.25 + 8.45 = e4, 894.62
12
Now, we calculate the TAEG rate, denoted as iT AEG :
For the calculation of the TAE rate, denoted as iT AE , we get a new F V 0 = F V − T = 4, 894.62 −
12.42 = e4, 882.20, where T = 4, 500 × ( 12
9
)0.092(0.04) = 12.42. Then we equate and calculate
24
We note that for the banker the TAE rate is an income rate, it cannot include tax, since tax is paid
by the customer, banker gets it and transfers to the tax authority.
In Section 2.2 we worked the Discount Rate, although it is an interest rate but of a different nature,
a discount interest rate. We won’t repeat and refer to the section. We can talk on other sorts of
rates, that although mathematically are worked like interest rates in Compound Interest, they
are of different nature also. We talk on cost, income and return rates on investments and other
applications. These are rates that may include interest, other costs as well as benefits, buy and
selling prices. We deal them in the applications dealt from Chapter 8 on. In Chapter 10 we define
Rate of Return on an Investment, that is going to be used also in Chapter 11 about shares.
To finish this chapter on rates we present the limit of the Annual Nominal Rate as the number of
conversion periods m tends to infinity, which we denote as δ = i(∞) = limm→∞ i(m) . This constant
is known as an Interest Force and is used in Continuous Compounding.
Definition 4.10 (Interest Force) Let us consider interest compounding, the Annual Nominal
Rate i(m) and that we can divide the year as many conversion periods as possible. δ = i(∞) =
limm→∞ i(m) is defined as the annual (constant) Force of Interest, where iA ≥ 0 is the effective
annual equivalent rate, i.e., such that
(m) m
!
iA
1 + iA = 1 + . (4.2)
m
Result 2 Let us consider interest compounding, the annual nominal rate i(m) with m conversion
periods, the effective annual equivalent rate iA , and the interest Force δ as defined in 4.10. δ exists
and Then
(∞)
δ = iA = ln (1 + iA ) .
(m)
Proof. Retrieve (4.2) and equate with respect to iA :
(m) m
!
iA
1 + iA = 1+
m
h i
(m)
iA = m (1 + iA )1/m − 1
h i
(1 + iA )1/m − 1
(m)
iA = 1
m
(m)
Compute the limit, limm→∞ iA , directly we have an indetermination:
h i
(1 + iA )1/m − 1 0
(m)
lim i = lim = .
m→∞ A m→∞ 1 0
m
25
Now, using l’Hôpital’s rule, we get
h i0 1
0
(1 + iA )1/m − 1 e m ln(1+iA ) − 1
(m)
lim i = lim 1 0
= lim
m→∞ A − m12
m→∞ m→∞
m
h 1
i
e m ln(1+iA ) ln (1 + iA ) − m12
= lim
− m12
m→∞
= lim (1 + iA )1/m ln (1 + iA ) .
m→∞
Hence,
(∞)
iA = δ = ln (1 + iA ) .
eδ = (1 + iA ) ,
so that we can find new formulae for Future and Present Value Formulas in Continuous Compound-
ing, next.
FV = P (1 + iA )t ,
FV = P eδ t ⇔ P = S e−δ t
Example 4.4 Mr. Zappa did an investment of e10 000 for t = 5 years with an interest force of
5%. Calculate the Future Value of the investment and compute the APY of the investment.
4.1 Exercises
1. True/False questions:
(4)
(a) Interest rate iA /4 is an effective quarterly rate equivalent to the respective annual rate.
T F
(m)
(b) Interest rate iA divided by m corresponds to the effective accumulation rate of some
principal.
T F
26
(c) Consider simple interest, and an interest rate fixed and positive. The nominal annual
interest rate, with quarterly accumulation, and positive, is always equivalent to the
corresponding effective rate.
T F
(d) In simple interest, nominal and effective interest rates are always equivalent rates.
T F
(e) In compound interest, the effective monthly interest rate is proportional to the (annual)
nominal rate.
T F
(f) Two rates of interest are said to be equivalent if they result in the same accumulated
value at any, and the same, point in time, no matter the interest regime.
T F
(g) Both in simple and compound interest, an annual nominal interest rate, positive and
compounded quarterly, can never be equivalent to the corresponding effective rate.
T F
(h) From the quarterly effective (and positive) interest rate were calculated both the annual
nominal rate and an annual equivalent rate. They are always different, no matter the
interest regime considered.
T F
(i) Let two non-negative interest rates referred to the same period, such as i1 > i2 . Then,
(m) (m)
the corresponding (annual) nominal interest rates also verify i1 > i2 .
T F
(j) For compound interest and a positive interest rate, the effective monthly interest rate is
proportional to the annual nominal rate with quarterly accumulation.
T F
(k) For both compound and simple interest, and a positive interest rate, the effective quar-
terly interest rate is proportional to the annual nominal rate with quarterly accumula-
tion.
T F
(a) Consider simple interest. Ms. Zilda wants to calculate the annual nominal rate corre-
sponding to the effective quarterly interest rate of 1%. The right answer is:
1. 2.01% × ; 2. 4.00% × ; 3. 4.02% × ; 4. 2.00% × .
(b) Consider compound interest. Zippo wants to calculate the effective rate (to the accumu-
lation period) corresponding to the nominal annual rate of 12% compounded monthly.
The right answer is:
1. 1% × ; 2. 0.95% × ; 3. 1.20% × ; 4. 0.1% × .
(c) Assuming a effective monthly rate of 0.5%, compute the equivalent effective rate for a
period of nine months under compound interest.
27
1. 4.591% × ; 2. 4.500% × ; 3. 0.667% × ; 4. Other × .
(d) Mr. Zed is considering some investments that accumulate monthly, corresponding to an
equivalent annual rate of 3.5%, under compound interest. What is the respective annual
nominal rate?
1. 3.557% × ; 2. 3.5% × ; 3. 3.445% × ; 4. Other × .
(e) Dr. Zen made a four year application of amount e 5, 000.00, under compound interest. It
accumulated at maturity an amount of e 6, 281.78. Determine the quarterly interest rate
that would produce the same accumulated amount with the same term (approximately).
1. 1.43% × ; 2. 1.60% × ; 3. 1.46% × ; 4. Other × .
(f) Consider compound interest. Calculate the annual nominal rate, compounded twice a
year, corresponding to a quarterly effective interest rate of 2%.
1. 8.08% × ; 2. 8.00% × ; 3. 4.04% × ; 4. 4.00% × .
(g) Mrs. Zen will be receiving e 5, 000, e 10, 000 and e 15, 000 within 6, 8 and 10 months,
(12)
respectively. Considering simple interest and an annual nominal interest rate of 7.8A %,
the Present Value of the total amount is (approximately):
1. e 30, 000 × ; 2. e 28, 402.53 × ; 3. e 28, 363.21 × ; 4. Other × .
(h) In compound interest, Mr. Zen did a four year application of e 5, 000 at a nominal
(annual) rate of 4% compounded quarterly. Write the interest earned in the first year
(approximately, rounded):
1. e 200.00 × ; 2. e 203.02 × ; 3. e 51.52 × ; 4. Other × .
(i) Mr. Zen will be receiving e 5, 000, e 10, 000 and e 15, 000 within six, eight and ten
(6)
months, respectively. Consider compound interest and interest rate of 3.9A %. The
present value of the total amount is (approximately):
1. e 30, 000% × ; 2. e 28, 402.53 × ; 3. e 29, 169.79% × ; 4. Other × .
(j) Consider compound interest. Choose the quarterly rate that is equivalent to the rate
corresponding to the annual nominal rate of 12%, compounded semi-annually:
1. 2.96% × ; 2. 3% × ; 3. 3.14% × ; 4. 5.83% × .
(k) Consider compound interest. Mr. Zappa is getting a bank loan and is being offered the
(12)
interest rate iA = 6%. The pair of “effective and equivalent rates is, respectively (give
the right answer, approximately):
(12) (12)
1. iM = 0.5% & iA = 6% × ; 2. iA = 6.17% & iA = 6% × ;
3. iM = 0.5% & iA = 6.17% × ; 4. i = 0.5% & i0 = 0.49% × .
(l) In compound interest, Mr. Ben did a four year application of e10, 000 at a nominal
(annual) rate of 2% compounded semi-annually. Write the interest earned during the
second year, approximately.
1. e 101.00 × ; 2. Other × ; 3. e 205.04 × ; 4. e 103.03 × .
(m) Consider compound interest and quarterly effective interest rate of 1%. Help Mr. Ed
select one of the following receipt options:
28
i. Receive e10, 100 in a year from now, and the same value within two years exactly;
ii. Receive e10, 000 in half a year from now, and the same value within a year from
then, exactly.
29
Chapter 5
Equation of value
Money and interest come always together, if you borrow money the longer you take to repay the
more you expect to spend on interest, given some positive interest rate. This means that money,
time and interest rate matter for money valuation. We mean, e1, 000.00 today does not mean the
same as e1, 000.00 within one year, and how much is the difference depends on the interest rate
you apply.
To compare financially two or more amounts of money due at different dates you need a mathe-
matical equation called Equation of Value.
Definition 5.1 (Equation of Value) Equation of Value is a mathematical expression that equates
several pieces of money that are due at certain dates, at a same chosen date called Focal Date or
Valuation Date.
Definition 5.2 (Equivalent Principal) For a given interest rate, two or more Principals are
said to be Equivalent if they report the same value at some focal date.
For a given interest rate, to evaluate the sum of a set of Principals you need to specify a focal date,
and you expect to have different values if you change either the Focal Date or the Interest Rate,
or even both. Furthermore, you may be able to find a point in time that equates two different
Principles. That is called Equivalent Time. We can define it as follows.
Definition 5.3 (Equivalent Time, Average Due Date) An Equivalent Time, or Average Due
Date, is a reference date where a single payment equals the sum of some original debts.
This definition can be generalised to compare a set of principals to any other set, with several
due dates. We should understand that when setting an Equation of Value is depending on the
Interest Regime, since they accumulate, or discount differently. The concept of Discounting we are
talking is the opposite operation to Accumulation. Accumulation is computing the Future Value,
Discounting is calculating the Present Value.
Example 5.1 Ms. Zita has scheduled two payments of e500.00 that are due within five and 12
(12)
months, exactly. They were calculated for an interest rate of iA = 6%. See Figure 5.1. Consider
the following questions:
30
1. What is today’s debt?
2. Ms. Zita would like to replace those two scheduled payments by one single payment of amount
e1, 000.00. Calculate the Equivalent Time.
Solution: First, remark that it is obvious that the solution of any of the questions will depend on
the interest regime. The term was set short to make it practical for single and compound interest,
although mathematically it could be any term. Effective rate is i(12) /12 = 0.5%.
1.
2.
The common least time is unit for these sort of operations is the day, the solution is 8
months and 14 days.
(b) In compound interest we have
The final answer corresponds to 8 months and 15 days. We know that in banking oper-
ations any fraction of a day count as a complete one.
500 ↓ 500
0 5 t 12
In the previous example we see how it may be possible negotiate a debt changing times of payment.
Of course it will also be possible to change payments amounts. Always be certain that we can
have different solutions according to the interest system, the basis will be given by the regime in
previous use.
Next we show some common problems that appear in Future or Present value calculation, with
money applications and payments due in different time instants or periods,where Equation of Value
is used. Other problems are dealt in following chapters of this manuscript.
31
Problem 5.1 (Equivalent Principal Payment) Zita SA has a payment scheduled to be paid to
a supplier a year from now of amount e 1, 100.00. The company proposed to pay today e 1, 000.00
telling that was considering an interest rate i(2) = 10%. Evaluate the pros and cons of the offer.
Solution: Effective interest rate is iS = i(2) /2 = 5%.
We could make a similar reasoning if we have several Principal payments, as follows (only done for
Compound Interest).
Problem 5.2 (Equivalent Principal Payments) Zita SA has a set of amounts to be received
from a client scheduled as shown in Figure 5.2, where unit time is the month.
Zita SA proposed the client to pay all amounts in Month 9, from now. With a rate of i(12) = 6%,
and Compound Interest calculate the single payment, denoted P9 .
Solution: Effective interest rate is iM = i(12) /12 = 0.5%, we have to find the Equivalent Principal
Payment.
P9 = 500 (1.005)4 + 1 000 (1.005)−3 + 1 500 (1.005)−9 ' e2 929.38 ,
Note that the focal point in the equation is at instant “9”, there’s no need to calculate P V at “0”
and then calculate the F V at “11”, although we could do that and from there to accumulate or
discount to ‘9”. These would be unnecessary double calculation.
32
Problem 5.3 (Internal Rate of Return – IRR) Referring to the previous Problem , suppose
that Zita, SA counterproposes a single payment of e2 900.00 at Month 11. Calculate the rate, say
r, that corresponds to the Equivalent Payments.
Solution:
500 (1 + r)6 + 1 000 (1 + r)−1 + 1 500 (1 + r)−7 = e2 900, 00 .
We have an equation on v = (1 + r)−1 with higher degree that cannot be solved directly, only
numerically, with a pocket calculator or with computer software. One can use Excel Spreadsheet
function: IRR(range,guess). It needs a guess solution and we need to be careful since it is an
equation that can have multiple solutions, a bad “tip” may lead to an unrealistic (wrong) solution.
Solution is approximately r = IRR ' 1.27%.
Another problem that sometimes is of interest, is the calculation of average rates, when we have
in hand applications, even several, with more than one rate, so that we would like to convert
everything into a single rate application. Mathematically we are talking on calculating an Average
Rate.
Problem 5.4 (Average Rate) Ms. Zita has schedulled a set of applications of amounts e2, 500,
e3, 000 and e3, 500, to be made now, within one and two years, respectively. Application rates
vary, 3.5%, 3.25% and 3%, respectively. Ms. Zita wants all applied money to be available within 3
years, in order to face an expected amount.
Calculate a return rate of all applications, with interest compounding.
Solution: The return of all investment is indeed an average rate, a weighted geometric average rate
(if it were with simple interest it would be a weighted arithmetic average). Calculation as follows,
r denotes that rate.
F V = 2 500 (1.035)3 + 3 000 (1.0325)2 + 3 500 (1.03)1 ' e9 574.96 .
Now, equate
F V = 2 500 (1 + r)3 + 3 000 (1 + r)2 + 3 500 (1 + r)1 = e9 574.96 .
Solution is r∗ ' 3.3138% (it also gives two complex roots). We used Mathematica software, we
could also do it with IRR(range,guess) function.
5.1 Exercises
1. True/False questions:
(a) The “Equation of Value” is a mathematical expression which allows to relate several
amounts of principal, referred to chosen dates and given interest rates, and calculate the
whole value in a focal date.
T F
(b) In compound interest, if two principals, eA and eB, are equivalent, and if eA is equiv-
alent to a principal eC, then whatever the reference time is considered, eB is also
equivalent to eC.
T F
33
(c) The annual discount factor, usually denoted as v, is given by the same expression either
we consider simple or compound interest.
T F
2. Multiple choice-single answer questions:
(a) A principal of e5, 000 accumulating at fixed rate i, semi-annually and in simple interest,
accumulated for four years an amount of e6, 281.78. Determine the quarterly interest
rate that, applied during the same term but in compound interest, accumulates the same
amount (approximately).
1. 1.920% × ; 2. 1.970% × ; 3. 1.440% × ; Other × .
(b) Consider simple interest and monthly interest rate of 0.5%. Zef has to pay e300, e600
and e900 within 3, 6 and 9 months, respectively. After six months from start he has
not made any payment. At that time what is Zefs complete debt?
1. e1, 778.29 × ; 2. e1, 791.20 × ; 3. e1, 752.62 × ; 4. Other × .
(c) Consider compound interest and quarterly effective interest rate of 1%. You have to
select one of the following receipt options:
i. Receive e10, 000 in a year from now, plus the same value within two years exactly;
ii. Receive e10, 250 in half a year from now, plus the same value within nine months
exactly.
1. Choose i. × ; 2. Choose ii. × ;
3. Indifferent × ; 4. Missing information × .
(d) Ziggy made a deposit of e13 000 under simple interest for six years at semi-annual effec-
tive interest rate of 1.5%. The semi-annual interest rate with quarterly accumulation,
associated with a deposit of the same amount, for a period of three years, that produces
the same income is equal to:
1. 2.56% × ; 2. 3.00% × ; 3. 3.44% × ; 4. 5.00% × .
(e) Ziggy jr. made a deposit of e13 000 under compound interest for six years at semi-
annual effective interest rate of 1.5%. The semi-annual interest rate with quarterly
compounding, associated with a deposit of the same amount, for a period of three years,
that produces the same income is equal to:
1. 2.56% × ; 2. 3.00% × ; 3. 3.44% × ; 4. 5.00% × .
(f) Under simple interest, compute a sole (unique) annual interest rate that makes equivalent
the whole set of the following investments (assume that they are all done simultaneously):
i. A principal of e3, 000 for two years at an annual interest rate of 4%;
ii. A principal of e8, 500 for 18 months at an annual interest rate of 3%;
iii. A principal of e12, 450 for six years at an annual interest rate of 7%.
1. 6.86% × ; 2. 7.10% × ; 3. 6.26% × ; 4. Other × .
(12)
(g) Consider compound interest. Mr. Zappa invested e10, 000 at rate iA = 15%. What
is the application term necessary in order to increase by 50% the principal invested (in
years, approximately)?
34
1. 4.032 years × ; 2. 5 years × ;
3. 2 years 8 monhts and 20 days × ; 4. Other × .
(h) H2O Company did a deposit of e10 000 under compound interest during four years.
During the first two years, the annual interest rate was 4%, during the third and fourth
years, the rate was 4.5% and 5%, respectively. What is the annual average rate of this
application (approximately)?
1. 4.50% × ; 2. 4.40% × ; 3. 4.37% × ; 4. Other × .
(i) Mrs Zita made a deposit of e 1, 000 under simple interest, under the following quarterly
rates: 1st year: 1.5%; 2nd year: 3.0%; 3rd and 4th years: 3.5%. What is the average
quarterly interest rate?
1. 2.875% × ; 2. 3.000% × ; 3. 2.556% × ; 4. 2.056% × .
Calculate:
(a) The amount produced by the first and second application at the starting date of the
third application.
(b) The constant annual interest rate that would produce the same accumulated value if the
company had only applied one sole amount of e150, 000.00, with a 54 month term and
compound interest.
35
Chapter 6
Annuities Certain
6.1 Definitions
In many situations people as well as investors face a given and regular sequence of payments during
some time horizon. The number of these payments can be of a predefined quantity or of a random
nature. In the first case we deal with Annuities Certain, it is the case of a sequence of loan payments,
a house rent, a set of periodic leasing payments. In the second case we have a Contingent Annuity,
it is the case of a life insurance. We are now going to deal with Annuities Certain only and in
countable time.
Example 6.1 A house rent, instalments of a loan, electricity or internet bills, etc.
The name Annuity links to the year, annual periods, however it generalised to any given accumu-
lation or conversion period, monthly, weekly or whatever.
Definition 6.2 (Term of an annuity) The Term of an annuity is the time of the whole opera-
tion, from start to end, the maturity.
10 20 50 60 10
0 1 2 5 6 t
There are many classes of annuities according to some property, we have already classified Certain
or Guaranteed annuities versus Contingent according to the knowledge of payment event, depending
on some sort of circumstance.
36
According to the term they can also be classified into temporary or perpetual. A Perpetuity is an
annuity with an infinite number of payments, like a Perpetual bond. In practice most are Temporary,
for instance the car Lease.
Dealing with the placement of the periodic payments, we can have an either an Ordinary Annuity,
same as Annuity-Immediate, versus an Annuity-Due. In the first case we have payments done
at the end of each time period and in the second each payments is done at the beginning of the
period. Figure 6.2 shows an example of a temporary annuity-due. According to deferment on first
20 10 50 60 10
0 1 2 5 6 t−1 t
Without loss of generality we now consider that an Ordinary Annuity has Unit Payments. We
study the computation of Present and Future Values with quick, closed formulae, that do not need
to be done stepwise. First consider the Present Value calculation, see Figure 6.3.
(P V )
1 1 1 1 1 1
0 1 2 5 6 t−1 t
Figure 6.3: Present Value for an Ordinary Annuity with unit payment
Formula 6.1 (Present Value of a unit payment annuity) Let P V = at i denote the present
value of a temporary, rate i > 0, unit payment annuity and maturity t. We have
1 − (1 + i)−t
at i = , i > 0. (6.1)
i
Proof.
37
with v = (1 + i)−1 . We have a geometric progression sum with rate r = v. Then we obtain, first
and last elements of sequence which are respectively u1 = v and ut = v t ,
u1 − ut × r v − vtv 1 − vt 1 − (1 + i)−t
at i = = = =
1−r 1−v 1/v − 1 i
If the unit of payment equals some constant R, its easy to get the general formula below.
Formula 6.2 (Present Value of a constant payment annuity, At ) Define At as the Present
Value of an Annuity where the unit of payment is R. Then we have
1 − (1 + i)−t
At = P V = R a t i = R , i > 0. (6.2)
i
For the calculation of the Future Value of a unit payment annuity we get Formula (6.3), see
Figure 6.2. From at i it’s easy to find formula for F V = st i .
(F V )
1 1 1 1 1 1
0 1 2 5 6 t−1 t
Figure 6.4: Future Value for an Ordinary Annuity with unit payment
Formula 6.3 (Future Value of a unit payment annuity) Let F V = st i denote the Future
Value of a unit payment annuity with maturity t. We have
(1 + i)t − 1
F V = st i = , i > 0. (6.3)
i
1 − (1 + i)−t (1 + i)t − 1
F V = st i = (1 + i)t = . (6.4)
i i
Formula 6.4 (Future Value of a constant payment annuity, St ) Define St as the Future Value
of an Annuity where the unit of payment is R. Then we have
(1 + i)t − 1
St = F V = R st i = R , i > 0. (6.5)
i
38
(P V )
1 1 1 1 1 1
0 1 2 5 6 t−1 t
Figure 6.5: Present Value for an Annuity Due with unit payment
Formula 6.5 (Present Value of a unit payment annuity due) Let P V = ät i denote the Present
Value of a unit payment annuity due with maturity t. We have, for t = 1, 2, . . . ,
setting that a0 i = 0.
1 − v t−1 × v 1 − vt 1 − vt
P V = ät i = = = (1 + i) = at i (1 + i)
1−v 1−v i
Remark 6.2 For a better understanding of Formula (6.6) consider in Figure 6.6 the representation
of an equivalent annuity (for the same rate i). Both give the same P V , since each unit payment is
accumulated (1 + i) by one time period. See Figure 6.5.
(P V )
1+i 1+i 1+i 1+i 1+i 1+i
0 1 2 5 6 t−1 t
For the calculation of the Future Value, see Figure 6.3, we get Formula that follows.
Formula 6.6 (Future Value of a unit payment annuity due) Let F V = s̈t i denote the Fu-
ture Value of a unit payment annuity due with maturity t. We have, for t = 1, 2, . . . ,
s̈t i = st i (1 + i) . (6.7)
39
(F V )
1 1 1 1 1 1
0 1 2 5 6 t−1 t
Figure 6.7: Future Value for an Annuity Due with unit payment
Proof. Immediate,
In an annuity sometimes the first payment for several reasons is delayed relative when some loan
contract is signed. Sometimes to give an extra time to investors to start recovering the money
spent. In this case we talk about a deferred annuity. See the example referring Figure 6.8.
(P V ) (F V )
500 500 500 500 500 500
0 1 2 3 4 5 6 7 8
Definition 6.3 (Deferred Annuity) It’s an annuity under which the first payment occurs at
some specified future time.
Formula 6.7 (Present Value of a Deferred Annuity) The Present Value, P V , of a deferred
annuity, with deferment k years, constant payment R and term t + k, is denoted as R k| at i and is
given by
PV = R k| at i = R v k at i .
40
(P V ) (F V )
1 ... 1 1
0 1 k k+1 k+t−1 k+t
Example that follows shows the calculations of Present Values and Future Values of an annuity in
presence and absence of a deferment.
Example 6.2 (Ordinary vs Deferred) The ordinary annuity figured below is compared against
the two year deferred one in Figure 6.8.
(P V ) (F V )
500 500 500 500 500 500
0 1 2 3 4 5 6
6.5 Perpetuities
Definition 6.4 (Perpetuity) A Perpetuity is an annuity with infinite term. They have applica-
tion in share valuation, for instance.
Present Value is easy to compute, just calculate the limit a∞ or ä∞ , respectively ordinary or due.
1 − (1 + i)−t
1 1 1
a∞ = lim at i = lim = lim 1− = , i > 0.
t↑∞ t→∞ i t→∞ i (1 + i)t i
41
Example 6.3 Perpetua will start soon her studies at ULisboa. After completing the course she
intends to stay and find a job nearby. She is undecided about accommodation, considering two
options:
a) Rent a Flat for e500.00 monthly; or, b) Buy a small flat, for that she needs a bank loan of
(12)
e120, 000.00. For a nominal rate of iA = 6% help Perpetua make a decision.
The effective rate is iM = 6%/12 = 0.5%. Let’s compute the present value of all the money she
could spend if she rents for a long time:
A main problem involving annuity calculation is to compute the common annual payment. As
usual, we write R as the periodical constant payment, i the interest rate, At = R at i as the Present
Value of the annuity, St = R st i as the Future Value of the annuity.
Problem 6.1 (Find Payment) Once known the Present Value, At with maturity t e a taxa i,
we equate
At
R= .
at i
∗
t = − ln (1 − i At /R) / ln(1 + i) .
i At = R 1 − (1 + i)−t
(1 + i)−t = 1 − i At /R
−t ln(1 + i) = − ln (1 − i At /R)
t = − ln (1 − i At /R) / ln(1 + i)
Problem 6.3 (Calculation of Rate i) Knowing At , R and t, similarly as above use equation
i At = R 1 − (1 + i)−t ⇔ (1 + i)−t = 1 − i At /R
It can be solved numerically with software: For instance, with Excel spreadsheet we can use functions
fx Rate and Solver (exact commands may depend on Excel version). Pictures in Figure 6.11 show
an example done with Excel for Mac 16.54 2021. Chan and Tse (2017) has a similar example
with MS Windows Excel.
42
Nper: Required. The total number of annuity payment periods
Pmt: Required. The constant payment
Pv: Required. The P V
Fv: Optional. The F V . If omitted, it is assumed to be “0”
Type: Optional. “0” (if omitted) or “1” with due payments
Guess: Optional. Starting value, 10% if omitted.
Output: Intrest rate i.
43
6.7 Exercises
1. True/False questions:
(a) An annuity is defined as a set of periodic constant payments, in equally spaced periods,
and where a constant interest rate is applied.
T F
(b) The term of an annuity is the same as its corresponding life.
T F
(c) A Pension payment is an example of a contingent annuity.
T F
(d) Consider an annuity due and an ordinary annuity with equal unit payments, same num-
ber of payments and term, corresponding the same constant and non-negative interest
rate.
The value ä10 corresponds to a10 accumulated one time period.
T F
(e) A sequence of periodic payments, located at equal time intervals, makes an annuity if
and only if all payments are equal.
T F
(f) “The discounted value of an annuity certain should be calculated with the simple discount
rate”.
T F
(g) Notation a10 8% stands for an ordinary unit payment annuity with 10 equal payments
where a constant interest rate of 8% is applicable.
T F
(h) Let i > 0 and n denote a constant interest rate and the term of an annuity certain,
respectively. Then, 1 + i s10 i = (1 + i)10 .
T F
(i) For rate i > 0, we have that st i > t, t = 2, 3, . . .
T F
(j) Notation ät i denotes an annuity due with t constant payments all equal to R, for a
constant interest rate i.
T F
(k) For rate i ≥ 0, we have that at i ≤ t.
T F
(l) For all t, t = 2, 3, . . . , and i ≥ 0 we have that st i = st−1 i + 1.
T F
(m) An unit annuity-due deferred one period is equivalent to an unit ordinary annuity.
T F
(n) ä10 5% = (1.05) a10 5% .
T F
44
(o) A set of consecutive, constant and equally spaced, payments is not an annuity if period
payment is monthly.
T F
(p) ä10 10.25% /a10 10.25% = (1.05)2 .
T F
(q) If a∞ i = 22, then i < 5%.
T F
(r) For all i and t, st i = v −t a t i.
T F
(s) Keeping i and t unchanged, the future value of an annuity, st i , does not change if the
annuity is deferred.
T F
(t) Let a∞ i = 20. Then we have at i /ät i = 1.05.
T F
(u) With a constant interest rate i, the present value of an unit perpetuity-due equals 1+1/i .
T F
(v) Let an annuity and the corresponding interest rate i ≥ 0. We have that
1/at i − 1/st i = i .
T F
(w) Let i > 0 and t denote respectively a constant interest rate and the term of an annuity
certain, whose future value is st i . Then,
(1 + i)t − 1
i= .
st i
T F
2. Multiple choice-single answer questions:
(a) Zach is going to receive 12 equal monthly amounts of e200.00 each, at the end of each
month, counting from today. For a monthly effective rate of iM = 1%, calculate the
present value of the annuity (approximately).
1. e2, 228.73 × ; 2. e2, 251.02 × ; 3. e2, 400.00 × ; 4. Other × .
(b) ZY X Company deposits every quarter e100 during six years in a bank account under
compound interest. Assuming a quarterly effective interest rate of 1.0%, the accumula-
tion value of this financial investment will be:
45
1. e2, 228.73 × ; 2. e2, 251.02 × ; 3. e2, 400.00 × ; 4. Other × .
(d) Mr. Zach sold his car to his friend Zappa for e12, 000. Zach received immediately e3, 000
and the rest is going to be received through 10 equal instalments, at the end of each
consecutive quarter. The first payment is to be done nine months after the transaction,
exactly.
For iT = 1%, calculate the value of each constant payment (approximately):
1. e969.34 × ; 2. e1, 455.50 × ; 3. e950.24 × ; 4. Other × .
(e) A laptop computer has a selling price of e1, 957.60 (after a certain discount). Mr. Zed
has been given the option to pay the computer in six monthly instalments of e332
(approximately) each, being the first one within six months. Considering a annual
interest rate of 6% compounded monthly, advise Mr. Zed the best option:
1. Pay e1, 957.60 immediately × ; 2. Pay the annuity × ;
3. Indifferent × ; 4. Not enough information × .
(f) A laptop computer has a selling price of e1, 321.91 (after a certain discount). Mr. Zen
has been given the option to pay the computer in six monthly instalments of e231 each,
(6)
where the first one is due within six months. Consider an interest rate of iS = 3%.
Give your (most sensible) choice:
46
(j) Mr. Zen faces an annuity whose value at some instant in time is 1050 s5 8% (1.08)−5 . He
is considering the calculations:
i. 1050 a5 8% ;
ii. 1050 ä5 8% ;
iii. 525 3| a2 8% + 1050 a3 8% + 525 s2 8% (1.08)−5 .
The above calculations are equivalent to 1050s5 8% (1.08)−5 , choose the best/complete
option:
1. i. × ; 2. ii. × ; 3. iii. × ; 4. i. and iii. × .
20 5 20 5 20 5 20 5 20 5
0 1 2 3 4 5 6 7 8 9 10 11 quarters
(a) Calculate the present value, using formulae referred to annuities, function of at i or st i ,
or both, for a semi-annual interest rate of 3%;
(b) Calculate its accumulated value at maturity, for the same interest rate;
(c) Calculate in Quarter 7, immediately after payment, the amounts due and overdue.
4. Zarco intends to get a loan in the ZeroZeroZeven Bank worth e200, 000.00. The annual
nominal rate is 6, 0% with monthly compounding. The term of this loan is 25 years and is
repayable with monthly constant payments (principal plus interest) at end of the month.
5. Dr. Zen agreed to sell his older vehicle to his friend Dr. Zaid for amount e12, 000.00. The
agreement determines that Dr. Zen will receive today a down payment of e3, 500.00, plus ten
equal instalments at the end of each quarter. Consider an effective annual rate of 10.9%.
(a) The first of the ten periodic payments is due within three months. Calculate the value
of each instalment.
(b) The e3, 500.00 down payment can be suppressed if the periodic payments are due at the
beginning of each quarter. Calculate the amount of each instalment under these new
conditions (no change in the interest rate).
(c) Get back to the initial situation in item (a). Due to unexpected financial troubles,
Dr. Zaid concluded that he is not going to be able to pay the last instalment. Dr. Zen,
who is a kind young man, understood the situation and forgave the last payment. In
this case, what was the value of the discount (to the initial sell agreement) that Dr. Zen
granted to his friend, referring today (date of the agreement)?
47
6. Mr. Zappa’s company, ZY X Co., bought a factory that cost e 200, 000. To finance this
investment, ZY X Co. negotiated a loan with the Youth Bank. The loan conditions were as
follows:
(a) What is the value of each constant payment and what is the value of the first reimburse-
ment (amortization after 6 months)?
(b) What is the amount of the loan outstanding at 1 January 2015, immediately after the
payment on that period?
(c) On the 1/01/2015, the bank informed the customer that the interest rate would rise to
7.0% (nominal annual interest rate). Compute the value of the new payment for the
next semester.
(d) How much is the accumulated excess cost in situation (c), when compared to the previous
situation (a), at the end of the loan term.
7. Mr. Zink needs to get a new car. He agreed with the seller to pay the car through 60 monthly
payments of e 200.
(a) Determine the value of the car knowing that the seller is charging a nominal annual rate
(12)
of 18%A and Mr. Zink will do the first payment within six months.
(b) The seller had offered a second possibility: Pay the car through 60 monthly payments of
e200 at the beginning of each month, starting immediately, with the same nominal an-
nual interest rate, charging also a residual value of e400 payable with the final payment.
Determine the value of the car if Mr. Zink had accepted the offer.
48
Chapter 7
7.1 Introduction
Last chapter shows how to compute either Present Value or Future Value when we have a constant
amount of periodic payments. Often we have a variable sequence. If so, under some variable pattern
it is still possible to have short and closed formulae for quick calculation. It is the cases we studying
in this chapter.
We can think of payments increasing with a constant rate of inflation, for instance. Or, suppose
that you have a home mortgage where payments are set along with an estimated annual increase
with your salary. See Figure 7.1 and the example next inspired from Broverman (2010).
Example 7.1 Zorro wants to purchase a 10-year annuity with the first payment starting within
a year from now, exactly. The annuity will be valued at an effective rate of iA = 4%, and he
anticipates that the annual inflation rate is about 2.5% a year for the coming years.
If the first payment is established at e500.00, what is the present value of the annuity?
Solution: The sequence of payments is
1.025
which is a sum of a geometric progression with rate r = 1.04 . Then
" 10 # "
1.025 10
#
500 1 − 1.025
1.04 1 − 1.04
PV = = 500 .
1.04 1 − 1.025
1.04
1.04 − 1.025
49
(P V )
R R(1 + g) R(1 + g)2 R(1 + g)t−2 R(1 + g)t−1
0 1 2 3 t−1 t
Remark 7.1 In the Figure 7.1 above we have set that first payment is R, then last payment is
R(1 + g)t−1 . This could be greater, equal or smaller than the first one. Indeed, it’s easy to see that:
5. For g =< −1 payments have alternate sign and there is no relevant practical application.
PV = v × t , for i = g .
Proof. Set v = (1 + i)−1 , the Present Value of the annuity form a sum of a geometric progression
with rate (1 + g)/(1 + i) = (1 + g)v comes
with (1 + g)v 6= 1 ⇔ i 6= g.
When g = i it is straightforward that P V = v × t, from (7.1).
Remark 7.2 It is easy to see that i < g both numerator and denominator are negative, then the
fraction is always positive.
50
Formula 7.2 When the first payment is R we have immediately,
t
1+g
1 − 1+i
PV = R , i 6= g , t = 1, 2, . . . . (7.2)
i−g
In the case of i > g, from the above expression we find easily the present value of a perpetuity whose
first payment is R, and growth rate in geometric progression. The present value of an annuity with
infinite payments corresponds to a geometric series with starting value equal to R/(1 + i) and rate
(1 + g)/(1 + i). See upcoming formula.
Example 7.2 (Payments in Increasing Arithmetic form) Dr. Zorro is buying a new TV set,
high resolution, he was given the option to pay in three instalments: e500, e600, e700 and e800,
interest is considered to be included at an effective rate of 5%. Calculate the present value.
Solution: Payments are disposed in an arithmetic progression, with growth 100. We have few
payments we can easily calculate P V , stepwise:
P V = 500 (1.05)−1 + 600 (1.05)−2 + 700 (1.05)−3 + 800 (1.05)−4 ' e2, 283.26 .
Or, we could try another way, as follows:
PV = 500(1.05)−1 + 600(1.05)−2 + 700(1.05)−3 + 800(1.05)−4 =
= 400(1.05)−1 + 400(1.05)−2 + 400(1.05)−3 + 400(1.05)−4 400a4 5% +
+ 100(1.05)−1 + 100(1.05)−2 + 100(1.05)−3 + 100(1.05)−4 100a4 5% +
+ + 100(1.05)−2 + 100(1.05)−3 + 100(1.05)−4 1001| a3 5% +
.
+ 100(1.05)−3 + 100(1.05)−4 1002| a2 5% +
+ 100(1.05)−4 1003| a1 5%
= 400a4 9% +
+ 100a4 5% + 1001| a3 5% + 1002| a2 5% + 1003| a1 5%
51
Then,
P V = (500 − 100) a4 + 100 a4 5% +1| a3 5% +2| a2 5% +3| a1 5% = 400 a4 + 100 (Ia)4
1 − v4 1 − v3 1 − v2 2 1 − v 3
(Ia)4 = + v+ v + v
i i i i
1 ä − 4 v 4
= (1 + v + v 2 + v 3 + v 4 − 4v 4 ) = 4
i i
Notation (Ia)t i is international, it denotes the the Present Value of a t-term annuity in Increasing
Arithmetic Progression with rate 1, starting payment equal to 1, and interest rate i.
We can write a general formula, just generalising expression in the example above, and/or prove it
in another way, mathematically more elegant.
Formula 7.4 Let (Ia)t i denote the P V of a t term annuity in Increasing Arithmetic Progression
with Rate 1, first payment 1, and interest rate i. We have that
ät − t v t
(Ia)t i = (7.4)
i
Proof.
(Ia)t = v + 2v 2 + 3v 3 + · · · + (t − 1) v t−1 + t v t
(1 + i) (Ia)t = 1 + 2v + 3v 2 + · · · + t v t−1 .
ät − t v t
(Ia)t = .
i
Formula 7.5 (General Formula for Increasing Arithmetic Annuity) Let R1 denote the first
payment of a t-term annuity in Increasing Arithmetic Progression with Rate h, and interest rate i.
The Present Value is given by
• If R1 = h then P V = h (Ia)t i
52
7.4 Annuities with payments in Decreasing Arithmetic Progres-
sion
Example 7.3 (Payments in Decreasing Arithmetic form) We refer to Example 7.2 and sup-
pose that Dr. Zorro was given the option to invert the order of the periodic payments, keeping the
interest rate: e 800, e 700, e 600 and e 500.
Calculate the present value.
Solution: Payments are disposed in an arithmetic progression, but with rate h = −100, or decreasing
with rate h∗ = 100, so that directly the P V , stepwise, is:
P V = 800 (1.05)−1 + 700 (1.05)−2 + 600 (1.05)−3 + 500 (1.05)−4 ' e 2, 326.48 .
Or, similarly as in Example 7.2 but starting backwards, with v = 1.05−1 , as follows:
PV = 800 v 1 + 700 v 2 + 600 v 3 + 500 v 4
= 400 v + 400 v 2 + 400 v 3 + 400 v 4
+ 100 v + 100 v 2 + 100 v 3 + 100 v 4
+ 100 v + 100 v 2 + 100 v 3
.
+ 100 v + 100 v 2
+ 100 v
= 400a4 5% +
+ 100a4 5% + 100 a3 5% + 100 a2 5% + 100 a1 5%
Then,
P V = (500 − 100) a4 5% + 100 a4 5% + a3 5% + a2 5% + a1 5% = 400 a4 + 100 (Da)4 ,
where (Da)4 corresponds to the expression inside brackets. (Da)4 can be simplified as follows,
1 − v4 1 − v3 1 − v2 1 − v 4 − a4
(Ia)4 = + + + = .
i i i i i
Notation (Da)t i is international, it denotes the Present Value of a t term annuity in Decreasing
Arithmetic Progression with Rate 1, last payment 1, and interest rate i.
A general formula for (Da)t i is given below.
Formula 7.6 Let (Da)t i denote the P V of a t term annuity in Decreasing Arithmetic Progression
with Rate 1, Last payment 1, and interest rate i. We have
t − at i
(Da)t i = . (7.6)
i
Proof. It can be done using the same method of (7.4).
Formula 7.7 (General Formula for Decreasing Arithmetic Annuity) Let Rt be the last pay-
ment of a t term annuity in Decreasing Arithmetic Progression with Rate h∗ , and interest rate i.
The Present Value is given by
P V = (Rt − h∗ ) at i + h∗ (Da)t i , (7.7)
where (Da)t i is given by (7.6).
53
7.5 Exercises
1. True/False questions:
(a) A set of periodic payments, no matter constant or variable, with constant or variable
interest rate, immediate or deferred, can build an annuity if they are equally spaced
along time.
T F
(b) Consider Formula (7.1). Examining the denominator, i.e., difference (i − g), if g > i then
we can conclude that P V < 0.
V F
2. Multiple choice-single answer questions:
(a) Dr. Zen is due to receive 10 monthly amounts, increasing at a monthly rate of 1% in
geometric progression. First payment is planned to be received within a month from
today, with a value of e10.00. For a monthly effective rate of iM = 1%, the present
value of the set of amounts is given by (in e):
(b) Consider the following periodic sequence of payments beginning at moment 1: 1000,
2000, 3000, 1000, 1000. The expression that determines the present value of those
amounts, referred at time 0, is:
1. 1000 a3 i + 2000 (1 + i)−2 + 3000 (1 + i)−3 × ;
2. 1000 (Ia)3 i + 1000 a2 i × ;
3. 1000 + 2000 (1 + i)2 + 3000 (1 + i)3 + 1000 (1 + i)4 + 1000 (1 + i)5 × ;
4. 1000 (Ia)3 i + 1000 (1 + i)−4 + 1000 (1 + i)−5 × .
(c) An annuity with payments varying in geometric progression and whose present value is
computed by the expression
1 − (1 + i)−2
R× ,
i
has a number of payments equal to t and growth rate g:
1. t = 1 and g = 0 × ; 2. t = 2 and g = 0 × ;
3. t = 2 and (1 + g) = 0.1 × ; 4. t = 2 and g = 1 × .
(d) Zulmira SA intends to buy a new equipment through a series of 10 annual payments.
Assume an ordinary annuity due. The payments will grow geometrically, with rate equal
to 1.1 and starting payment equal to e10, 000.00.
Compute the present value of the equipment, assuming an annual effective rate of 5.0%.
1. e118, 466.57 × ; 2. e124, 389.90 × ; 3. e112, 825.31 × ; Other × .
(e) Zeferino Zacarias, a student at ISEG, needs your help on the following problem:
What is the increasing rate of an annuity in geometric progression with three
payments, where the last payment doubles the second, and the first is half of the
second?
54
1. 3 × ; 2. 1/2 × ; 3. 2 × ; 4. 1 × .
3. Zeca Ltd is buying a new exercise equipment for the gym. With a 2% effective monthly
interest rate and instalments to be due at the end of each month, compute the price value of
the equipment (at buy) for the following choices (keep the sequence order):
55
Chapter 8
Loan Repayment
8.1 Preliminaries
We can have several sorts of loans and their repayment. Often short time loans involve smaller
amounts and often are repaid entirely once at the maturity. Interest can be paid upfront or at
maturity together with the repayment. We talk about Bullet Loans. Medium and long term loans
involve higher amounts of money, repayments and interest are periodically paid along the term.
Principal is repaid by amortizations and interest is paid in conversion periods.
The last type of loans are the ones we are studying now. We will be building Amortization Schedules
where we book the evolution of the loan along its term, with interest payments and principal
repayments or amortizations. In particular we focus on loans with:
Constant Payments: A sequence of equal or level size payments per period, each composed of
the interest due plus a portion of the principal.
Constant Principal Amortization: Principal amortizations are constant throughout the life of
the loan. In this case Total payment, Principal plus Interest is non increasing with time.
For organizing/supervising a loan, economists build Amortization Schedules, where in every year/period
every Interest due, Amortization paid, as well as Outstanding Balance is registered. At maturity
Balance should be zero, exactly.
At time t, t = 1, 2, . . . , we denote the periodical Outstanding Balance as Bt , Amortization or
Principal Repayment as Rt , Total payment as Pt and Interest due as It , at the end of each period.
In Constant or level Principal, total Payment is decreasing, interest is also decreasing, since Amor-
tization is constant.
56
Example 8.1 (Level Principal Loan) Dr. Zoroaster contracted a 12-year loan of amount e 3, 000
with Level principal loan at an interest rate of 2%. Build the Amortization schedule.
Solution: We have Rk = 3000/12 = 250.00, Bk = Bk−1 − Rk , Ik = i × Bk − 1, and Pk = Ik + Rk ,
for k = 1, 2, . . . , 12. Then we get the Table 8.1 below
k Ik Rk Pk Bk
0 3000
1 60 250 310 2750
2 55 250 305 2500
3 50 250 300 2250
4 45 250 295 2000
5 40 250 290 1750
6 35 250 285 1500
7 30 250 280 1250
8 25 250 275 1000
9 20 250 270 750
10 15 250 265 500
11 10 250 260 250
12 5 250 255 0
Formula 8.1 (Amortization Schedule Calculation) Let B0 be the Principal or the amount of
a loan with term t. Then for k = 1, 2, . . . , t we have, in sequence:
The value of the loan at time “0” is always equal to the future instalments (Amortization + Interest)
discounted down to time “0”:
B0 = P1 v 1 + P2 v 2 + · · · + Pt v t . (8.1)
An example of this type are, often, long term loans like home mortgage schemes. Total Payment
is constant, or Level Payment, Interest slowly decreases, in early years it makes most part of
the payment, conversely, in late years amortizations makes most of it. Interest is decreasing but
Amortization is increasing.
Example 8.2 (Level Payment Loan) Dr. Zoroasters son, Zarathustra, contracted a 30-year
home mortgage plan of amount e100, 000.00 with Level Payment Loan, denoted as R, at an interest
(12)
rate of 6A %. Principal and interest to be paid monthly, so that term is 360 months. Build the
Amortization schedule.
57
Solution: In Table 8.2 we show a portion of the Amortization Table. First compute the Payment:
10, 000
10, 000 = P a360 0.5% ⇔ P = = 599.550 525 ,
a360 0.5%
k Ik Rk Pk Bk
0 100000.00
1 500.00 99.55 599.55 99900.45
2 499.50 100.05 599.55 99800.40
3 499.00 100.55 599.55 99699.85
4 498.50 101.05 599.55 99598.80
5 497.99 101.56 599.55 99497.24
···
358 8.90 590.65 599.55 1190.17
359 5.95 593.60 599.55 596.57
360 2.98 596.57 599.55 0.00
Figure 8.2 shows a graph with the behaviour of the Outstanding Balance
58
8.3 Common problems with amortization schedules
We can find the situation of the loan at some point, for instance immediately after the k−th
payment. For a t-term loan, that means it remains to be paid t − k payments. Or, the Outstanding
Balance at k-th year: Bk , k = 0, 1, . . . , t.
Finding the Outstanding Balance at k−year, immediately after the k-th payment, it is the same as
the Balance at the beginning of Period k + 1. See Figure 8.3. Recall that Pk = Rk + Ik , and that
Bk corresponds to all future due payments.
In a constant principal loan it’s immediate, it is not the case in the constant payment loan.
0 1 k k+1 t−1 t
Problem 8.1 (Find Bk , Level Principal Pay) Let Rk = R , ∀k = 1, . . . , t, then the Outstand-
ing Balance Bk is given by
Bk = B0 − k × R ,
and Interest to be paid in period k, Ik , comes
Ik = Bk−1 × ik ,
Problem 8.2 (Recursion for Bk , Level Principal Payment, Retrospective Method) Let Bk
be the Outstanding Balance at the end of period k of Loan B0 , with maturity t. For k = 1, 2, . . . , t
we have:
Bk = Bk−1 (1 + i) − Pk .
Problem 8.3 (Recursion for Bk , Level Principal Payment, Prospective Method) Let Bk
be the Outstanding Balance at the end of period k of Loan B0 , with maturity t. For k = t − 1, t −
2, . . . , 1, 0, we have:
Bk = (Bk+1 + Pk+1 ) v .
Example 8.3 (Level Principal Payment Loan, cont.) From Example 8.1, we calculate easily
the sequence through the Retrospective Method (output sequence is forwards) starting from B0 :
59
and through the Prospective Method (output sequence is backwards), starting from B12 ,
We can build any intermediate sequence as well, starting from a known value.
For loans with Level Total Payments we consider the following problems.
Bk = P at−k i .
Bk = B0 (1 + i)k − P sk i .
Problem 8.6 (Find Ik , Level Payment) Let Ik be the Interest due in year k, Rk the Principal
Repayment in year k, Pk = P = Ik + Rk the Level Payment, ∀k = 1, . . . , t, and constant interest
rate i. Then Ik is given by
Ik = Bk−1 × i = i × P at−k+1 i .
Example 8.4 From Example 8.2, Calculate Outstanding Balance B250 , I20 and I360 .
Solution :
We can notice in Table 8.2, as well as in the figures above, that in early years the amount of interest
due is quite a big portion of the yearly payment, and that how it contrasts with later years.
Now, considering loans with Level Payment we can establish recursions identical to those in prob-
lems 8.2 and 8.3.
Problem 8.7 (Recursion for Bk , Level Payment, Retrospective Method) Let Bk be the Out-
standing Balance at the end of period k of Loan B0 with maturity t. For k = 1, 2, . . . , t we have:
Bk = Bk−1 (1 + i) − P .
Problem 8.8 (Recursion for Bk , Level Payment, Prospective Method) Let Bk be the Out-
standing Balance at the end of period k of Loan B0 with maturity t. For k = t − 1, t − 2, . . . , 1, 0,
we have:
Bk = (Bk+1 + P ) v .
60
Example 8.5 (Level Payment Loan, cont.) From Example 8.2, we compute easily the sequence,
by the Retrospective Method (output sequence is forwards) starting from B5 :
B5 = 99, 497.24 → B6 = 99, 395.18 → B7 = 99, 292.61 → B8 = 99, 189.52 → B9 = 99, 085.92 . . . ,
and for the Prospective Method (output sequence is backwards), starting from B360 = 0,
We can build any sequence, either initial, intermediate or final, as long as we know the starting
value.
8.4 Exercises
1. True/False questions:
(a) In a constant and periodic repayment loan, both the principal and interest payments are
non-increasing along time.
T F
(b) In a long run loan repayment with equal principal payments and interest rate also con-
stant, along time, interest payments are also constant.
V F
(c) In a loan repayment with equal payments principal amortization is always constant.
T F
(d) Consider a 2-year loan with interest paid semi-annually and principal repayment at the
end. Total interest paid is I = P [(1 + iS )4 − 1].
T F
(e) A e100, 000 loan is redeemed five times with constant principal payments. None of the
individual (total) payments is higher than e20, 000.
T F
(f) A five year e100, 000 loan is redeemed four times with level principal payments of
e25, 000 each, with an annual interest rate of 5%. Interest payment in the last year
totals e1, 250.
T F
(g) The annual percentage rate (AP R) is the rate at which the cash value of a loan equals
the present value of all payments.
T F
(h) Consider a loan with cash value B(= B0 ), constant amortizations equal to R each, fixed
interest rate iA and term t years. The balance at the end of year k (1 ≤ k ≤ t) is equal
to (t − k) R.
V F
61
(a) Consider simple interest and a monthly rate of 1.2%. Mr. Zilch got a loan with cash
value of e57, 000.00. Principal was redeemed altogether once at the end, interest and
principal all summed e63, 839.54. What is the term of the loan? Approximately:
1. 1 year, 2 months and 3 days × ; 2. 9 months and 10 days × ;
3. 9 months and 15 days × ; 4. 10 months × .
(b) Consider that enterprise Zeta intends to get a loan contract, in which the principal is
paid at end of the contract and the interest at the beginning of the contract, that is
upfront. Loan term is one year, corresponding to an annual interest rate to be effectively
paid of 4% exactly.
Knowing that the principal repaid at the end of the year is e24, 000.00, what is the
amount of interest to be paid at the beginning?
1. e1, 810.65 × ; 2. e960.00 × ; 3. e923.08 × ; Other × .
(c) Consider a loan with cash value e100, 000, a term of 3 years, and semi-annual constant
repayments (amortizations). First repayment was done six months after signing the
contract, interest is paid semi-annually and the effective semi-annual rate is 5%.
What is the total payment due at the end of the first semester?
1. e21, 666.67 × ; 2. e23, 540.00 × ; 3. e21, 760.56 × ; Other × .
(d) Consider the following information about a constant payment loan (principal plus inter-
est, in e):
Debt at Cumulative Debt at
Period beginning Interest Payment Amortiz- Amortiz- end of
of period ation ation period
11 121, 878.92 731.27 3, 871.08 3, 139.81 6, 260.89 118, 739.12
The accrued amortization at end of period 12 is:
1. e10, 131.97 × ; 2. Other × ; 3. e12, 576.42 × ; e9, 419.54 × .
(e) Consider the following information about the amortization schedule of an existing loan,
corresponding to years k and k + 1, from Mr. Zeus’ company:
Debt at Cumulative Debt at
Year beginning Interest Payment Principal Amortiz- end of
of year paid ation year
k e99, 932.90 e666.22 e733.77 e134.65
k+1 e665.77 e733.77
Argue on the Amortization Method used, from those studied:
1. Amortizations are decreasing × ; 2. Amortizations are constant × ;
3. Not enough information × ; 4. Amortizations are increasing × .
(f) Consider the following amortization schedule of a loan (Dr. Zens) with annual constant
payments (principal plus interest) to be redeemed in ten years (in e):
Year B2 I3 R3 P3 Cumulative R3 B3
3 2, 604, 704 260, 470 227, 766 488, 236 753, 905 2, 376, 938
62
Principal Amortization in the last scheduled period is (approximately):
1. e511, 211 × ; 2. e488, 236 × ; 3. e443, 851 × ; Other × .
3. Ms. Zoom intends to contract with her bank a 15-year mortgage loan whose amount is
e125, 000. The bank offered a mortgage loan contract with the following conditions:
Calculate:
Month k Ik Rk Pk Bk
0 – – – 125, 000.00
1
2
3
4
4. Zone intends to contract a loan with Bank ZED of amount e200, 000. The annual nominal
rate is 6.0% with monthly compounding. The term of this loan is 25 years and it is payable
through monthly constant payments (principal plus interest) at end of each month.
5. Mr. Zappa intends to contract a loan to buy an equipment worth e100, 000. The bank offered
a loan contract with the following conditions:
63
(b) Compute the first five lines of the following amortization table:
Period Ik Rk Pk Bk
0 – – – 100, 000.00
1
2
3
4
5
6. Mr. Zen went to his bank intending to contract a 10-year mortgage loan of amount e100, 000.
The bank offered a contract under the following conditions:
(12)
• Annual interest rate: iA = 6%;
• Loan is payable through equal/level monthly payments (including interest) at the be-
ginning of each month, immediately due.
7. Mr. Zarco is buying a new car and requires a loan of e24, 000.00 to pay for it. A car dealer
offers two alternatives for the loan:
(i) Monthly payments for three years, starting one month after purchase, with an annual
interest of 12% compounded monthly; or
(ii) Monthly payments for four years, also starting one month after purchase, with annual
interest of 15% compounded monthly.
Denote by P1 and P2 the monthly payments for options (i) and (ii), respectively.
(a) Calculate P1 ;
(b) Calculate P2 ;
(c) Help Mr. Zarco to decide. Explain brief but clearly your option;
64
(d) Compute the first three lines of the Amortization Table corresponding to Option (i):
Time k Ik Rk Pk Bk
0 – – – 24, 000.00
1
2
3
(e) Compute the following lines of the Amortization Table corresponding to Option (ii):
Time k Ik Rk Pk Bk
45
46
47
48 0.00
65
Chapter 9
Leasing
Sometimes, it is better to rent an asset than buy it, or borrow money to buy. To Lease is a financial
option often in situation like: instead of borrow money to get an asset, borrow the asset, use it and
pay a rent.
Definition 9.1 (Leasing) A contractual arrangement that grants the use of a specific fixed asset,
for a specified time in exchange for payment, usually in the form of a rent, with a buying option,
later, for an agreed value called Residual Value.
There are several sorts of Leasing contracts, like for instance Operating Leasing and Financial
Leasing. The first is a short term cancellable arrangement and the latter being a long-term non
cancellable agreement.
The person or institution who receives the use of the asset under a lease is called the Lessee. He
uses the asset, not owns, and pays a rent.
The Lessor is the one that conveys the use of assets under lease. He owns, but not uses, the asset
and receives the rent.
Nowadays it is very popular to do a Lease, to individuals to companies, to institutions, instead of
buying a property or an equipment such as a car, an apartment, a business looking for an office
space or even a smaller machine like a printing system or photocopier.
Example 9.1 (Car Loan) Dr. Zbigniew is looking for a new car. Leasing is essentially the same
as renting the car for a set period of time. If he leases, he will have the right to drive the car, will
have the responsibility for maintenance and insurance, but he will not actually be the owner of the
car. At the end of the contract, periodic payments cease and the car may be returned to the leasing
company unless he buys for a Residual Value, previously established.
Dr. Zbigniew has the option to finance the entire cost of the car with a 5-year loan at an 7.2% annual
interest rate compounded monthly to be paid in equal amounts. Calculate the monthly (re)payment,
R, knowing that the entire cost of the car is e20, 000.
Solution:
20, 000 = R a60 0.6% ⇔ R = e 397.91
66
Example 9.2 (Car Lease) Dr. Zbigniew has also the choice of signing a lease, he doesn’t directly
borrow any money, but the car itself. With the lease he has to partly make monthly lease payments,
another part is repaid at the end of the lease by returning the car to the leasing company, and he
still has to do a downpayment upfront of 10% on the Asset Cost or Contract Value.
He still has the obligation to do on his own a proper maintenance of the vehicle.
The leasing company has determined that after two years the value of the car is e12, 000. This is
the car Residual Value after this period. Of the e20, 000 that Dr. Zbigniew borrowed in the form
of the car, minus the downpayment, he has to repay e12, 000 by returning the car. The remaning
(12)
cost value need to be covered by monthly payments, considering an interest rate of iA = 6%.
Find the monthly payments. Solution:
Example 9.3 (Car Lease, again) In continuation of Example 9.2: Alternatively, you could cal-
culate the Lease Payment R∗ splitting into Payment on Loss plus Interest on Residual. Payment
on Loss can be calculated as, Loss = 20, 000 − 2, 000 − 12, 000 = 6, 000:
Often, considering Lease Payments as ordinary annuity payments and Residual Value calculated
together with the last Lease Payment (the Residual Value can be deferred or split into more than
one payment, as long as the money value is calculated at that focal point last Lease Payment) we
can formulate a general leasing formula as below:
Formula 9.1 (Leasing Formula) Let AC denote the Asset Cost, DP the Down Payment (only
once), R the Lease Payment, RV the Residual Value, t the term and i a fixed interest rate, we
equate
AC = DP + R at i + RV (1 + i)−t
where RV is evaluated at focal point t.
67
9.1 Exercises
1. True/False questions:
(a) ZubiZevaNovi Ltd is preparing a lease contract to finance the acquisition of an equipment
that is attached to the following payments:
• Down payment of e100.00;
• 50 monthly constant payments at the beginning of each following month;
• Residual value of e100.00 (payable one month after the last instalment);
• Monthly effective rate is 1%.
What is the purchase value of the equipment?
1. e5, 200.00 × ; 2. e4, 080.42 × ; 3. e4, 079.81 × ; Missing info × .
(b) ZubiZarrEta SA is admitting a lease contract to finance the acquisition of an equipment
that is attached to the following payments:
• Down payment of e100.00;
• 50 monthly of e100.00 equal payments at the beginning of each following month;
• Residual value of e100.00 (payable one month after the last instalment);
• Monthly effective rate is 1%.
What is the purchase value of the equipment?
1. e5, 200.00 × ; 2. e4, 080.42 × ; 3. e4, 079.81 × ; e5, 000.00 × .
(c) Zoroaster Space Travel is acquiring a new vehicle through a leasing contract. The con-
tract defines: A down payment equal to e1, 500.00, eight quarterly level payments due
of e1, 500.00, and a residual value equal to the down payment value (paid 3 months after
the last due payment). The quarterly effective rate of this operation is equal to 1.5%.
The cash value of the vehicle is:
1. e12, 728.89 × ; 2. e14, 040.78 × ; 3. Other × ; 4. e14, 228.89 × .
68
(d) Zambrotta leased an equipment with purchase value of e12, 000. Down Payment and
Residual Value are e2, 400 and e600, respectively. Remaining quantity will be paid
through three annual level immediate (ordinary) payments. Residual Value is due with
the last periodical payment. Effective interest rate is 6% per annum.
At the time of the contract, calculate the total value to paid through those future
instalments.
1. e9, 000.00 × ; 2. e9, 096.23 × ; 3. Other × ; e14.231, 15 × .
(e) Ziva Ltd acquires a new equipment through leasing. Contract defines:
• A Down Payment equal to e1, 500.00;
• Eight quarterly constant payments due at end of quarter of e1, 500.00 each;
• Residual Value equal to the down payment, due three months after the last instal-
ment;
• Quarterly effective rate of the operation is 1.5% .
Compute the purchase price of the equipment.
1. e12, 728.89 × ; 2. e14, 040.78 × ; 3. e14, 228.89 × ; Other × .
3. Mr. Zed financed his car acquisition with a leasing contract under the following conditions:
Compute
(a) The instalment payments that are associated with the leasing contract;
(b) The global amount of all future periodic payments at the end of the first year, immedi-
ately after the corresponding payment.
4. Zach PLC is acquiring a motor car through a leasing contract. Contract value is e20, 000.
The company received the following proposal from LeasingAuto Ltd :
69
(b) Calculate the amount of the instalments still due one year after the contract date, im-
mediately after the corresponding instalment payment.
(c) In the meantime Zach PLC received another proposal from a different Leasing company,
with a quarterly interest rate of 2% and the following due payments:
• No Down Payment is due;
• Eleven quarterly instalments, immediate and growing in geometric progression with
rate 1.2, with the first to be due at the end of the first quarter;
• The Residual Value corresponds to the last instalment.
Compute the value of the first quarterly payment of this new proposal.
5. Dr. Zen is acquiring a SUV vehicle through a leasing operation. Purchase Value is e30, 000.
Dr. Zen received the following proposal from LeasingZappa, Ltd :
Calculate:
6. Mr. Zeus is going to get a brand new, luxurious, SUV (Sport Utility Vehicle) through a
leasing contract. It has a Contract Value of e 80, 000 and the following conditions:
Calculate:
7. Ms. Zaragatoa financed a car acquisition, a luxurious electric car (a Tres La), through a
leasing contract having the following conditions:
70
• A quarterly rate of 3.0% is effective in the first two years, and 3.25% afterwards;
• Constant payments every quarter (at the end of period);
• The first payment is to be done three months after the date of contract (no deferment);
• A down payment of 15% of the contract value is to be done upfront;
• A residual value of e8, 000 is due together with the last payment, and corresponds to
10% of the contract value.
Calculate:
(a) The contract value, the present value of both the residual value and the down payment
(at the date of the contract signature).
(b) The periodic level payments that are associated with the leasing contract.
(c) At the end of the second year, compute Ms. Zaragatoas payments due, immediately
before the corresponding instalment.
8. Business activities of Mr. Zappa are a major success. Unfortunately, the same cannot be said
of Zappas girl friend, Zubi, whose company is in pre-bankruptcy. Mr. Zappa intends to save
Zubis company. The main difficulty of Zubis company is the lack of liquidity caused by lease
payments.
The characteristics of the lease contract are as follows:
• Term: 24 months;
• Start date: 01/07/2015;
• Down payment: 10% of the contract value;
• Residual value: 2% of the contract value, payable with the last instalment;
• Contract value: e 50, 000;
• Lease payments: 24 monthly constant lease payments (at end of each month, it does not
include neither the down payment nor the residual value);
• Nominal annual interest rate of 12% with monthly accumulation.
9. A certain leasing contract consists of 55 monthly payments (payments are due). The first
payment occurs when the leasing contract is signed. The remaining 54 payments are of e100
each, and there is an added fee of e1.5 on each payment. The residual value is equal to e400
and occurs with the last payment. The agreed annual effective interest rate is 10%. Knowing
that the purchase value was e13, 500,
71
(b) A friend took over the leasing contract, but after the 24-th payment she faced financial
difficulties, stopped the temporary payments, restarting after 6 months only. The Leas-
ing company then changed the effective monthly interest rate to 1% (rate was changed
immediately after payments were reinstated).
Compute the new periodic payments.
72
Chapter 10
Bonds
10.1 Definitions
Definition 10.1 (Bond) A Bond is debt instrument issued directly to the public that pays interest
at regular intervals and that matures at some specific, given date in the future.
Bonds are a loan instrument, composed by small amount and indivisible units. A Bond issuance
represent a loan, often of big amounts as a whole, with big numbers of units, the bond. Bond terms
can be finite or infinite, defined at issuance.
Institutions that are allowed to issue bond loans are Public Companies and Governments. They
target the public directly, borrowing money, by selling these units or titles. Issuers, who sell, pay
interest, buyers or investors lend money, receive interest and are allowed to trade them in the
financial markets.
Issuers are borrowers and buyers are lenders under a public contract, available to public notice.
Public Companies issue bonds, only with the authorization of Financial Market Regulators or
Financial Control Bodies. Governments issue bonds as Public Debt, such as Treasury Bonds.
Buyers are Investers and Money Lenders. Public in general can buy bonds, individuals, organiza-
tions, companies, institutional funds, pension funds, insurers...
Some important technical concepts are to be learned, as they are part of the vocabulary in finance
world.
Definition 10.2 (Coupon) It is the periodic interest payment, at the conversion period and is
Guaranteed.
Definition 10.3 (Term) It is the life of a bond issuance. It can be finite, short or long,but it can
also be infinite or perpetual even. In the same issue, redemption dates are often different, fixed or
random, but at defined dates.
Definition 10.4 (Face/Nominal Value) The Face or Nominal Value is the value that is written
in the debt, unit, over which the interest is calculated.
73
Definition 10.5 (Issue Value) Issue Value is the value at which each bond is sold when issued,
or at issuance, not necessarily at the Face Value.
If sold at the issue value, one says that it’s issued or sold at the par; If sold with a discount then
issue value is below the par; Above the par otherwise.
Definition 10.6 (Bond Redeeming) It is the process called when the bond holder receives his
principal back at the redeeming date.
Definition 10.7 (Maturity) The bond holder receives his principal back at the maturity date. It
is normal that bonds have different redeeming or maturity dates. Or even no maturity dates at all
if they are randomly redeemed.
Definition 10.8 (Maturity/Redemption Value) The Maturity or the Redemption Value is the
value at the end of its term.
Like at issuance, Redemption Value can be at the par, above or below the par. If redeemed above
the par the bond is paid with a Premium.
Definition 10.9 (Market value) It is the value traded in the financial market.
Definition 10.10 (Bond Yield) In general, the Bond yield is the annual return an investor re-
alizes on a bond. It incorporates the coupon.
There may be other definitions for a Bond Yield. Another definition is the Quotient between the
annual interest paid (annual Coupon) and the bond value (Purchase, Market Value).
There are also several types of Bonds, apart from separating Sovereign Government Bonds and
Company Bonds. For instance, we can talk on:
Coupon Bonds These are unregistered bonds, no names are attached, also known as Bearer
bonds;
Mortgage bonds It is a bond secured by a mortgage, it comes with a fixed asset attached as
security;
Convertible Bonds Bond issued with an option given to convert the loan into shares. It pays the
fixed coupon but it can be converted into a predetermined number of equity shares during
the bond life at the discretion of the bondholder;
Zero Coupon Bond It is a bond that does not pay interest. Instead, it is traded at a deep
discount rendering a profit at maturity, where it is redeemed at the face value;
Index-linked Bond It is a bond which interest is related to some price index, for example Interest
indexed to inflation.
In the market Bond prices fluctuate according to day-to-day trading at the securities secondary
market, since they are negotiable.
74
10.2 Amortization Schedules
Like in Chapter 8, Bond issue is a loan too, one build Amortization Schedules to follow loan
repayment. This is necessary for taxation and other accounting purposes, it is important to follow
interest due payments as well as redemption payments.
Example 10.1 Zaratustra PLC, issued a bond loan in Euro currency with the maximum value of
e 200 000. From the Bond Term Sheet we took the following technical info:
• Coupon Payment: Payable semi-annually, first will occur one semester after issue;
• Redemption: Above the par. Semi-annually redemptions in equal number of bonds, starting
one year after issue date;
• Redemption Premium: e 0.25 per bond for the 1st and 2nd, e 0.50 for 3rd and 4th, and e0.75
for last.
Build the appropriate Amortization Table for the above bond loan.
Solution: Consider issue date as time 0, semi-annual periods counted as k = 0, 1, 2, . . . , 6, Out-
standing Balance as Bk , Interest due as Ik , Bonds redeemed as Nk , Amortization or Principal
Redemption as Rk , Premium as Πk and Total Payment as Pk . Lower letter πk counts for premium
per unit redeemed.
k Ik Nk Rk πk Πk Pk Bk
0 200, 000
1 6, 000 6, 000 200, 000
2 6, 000 4, 000 40, 000 0.25 1, 000 47, 000 160, 000
3 4, 800 4, 000 40, 000 0.25 1, 000 45, 800 120, 000
4 3, 600 4, 000 40, 000 0.50 2, 000 45, 600 80, 000
5 2, 400 4, 000 40, 000 0.50 2, 000 44, 400 40, 000
6 1, 200 4, 000 40, 000 0.75 3, 000 44, 200 0
75
10.3 Bond Valuation
We have defined Bond Yield, in Definition 10.10, as a particular rate of return. We can define more
generally a Rate of Return on an Investment.
Definition 10.11 (Return Rate on Investment) The Return Rate of an Investment it is the
rate (like an interest rate) that equates the whole Investment Outflow (Outcome) to the Inflow
(Income) properly discounted or accumulated to some fixed focal date, that it generates.
The calculation of the Return Rate uses an Equation of Value. Another particular Return Rate for
bonds can be defined as Yield to Maturity. As follows.
Definition 10.12 (Yield to Maturity (YTM)) Annual Rate of Return the buyer gets if he buys
and holds the bond until its maturity date when it is redeemed. In another way, it is the interest
rate that equates interest and principal payments to be received in the future to the present cost.
Also called the Effective Rate of Return.
Formula 10.1 (Calculation of Y T M ) Let j be the Y T M , P the Purchase value (Market value),
S the Redemption value, C = F r the annual Coupon payment (fixed), F the Face value, r the
Coupon Rate, vj = (1 + j)−1 the discount factor and t the Term. Y T M is the rate j such that
t
X C S
P = +
(1 + j)t (1 + j)t
k=1
= F r at j + S vjt . (10.1)
P = F + F (r − j)at j . (10.2)
Proof.
Since at j = (1 − vj t )/j ⇔ vj t = 1 − j at j , replacing in (10.1), with S = F , we get immediately
P = F r at j + F (1 − j at j ) = F + F (r − j)at j .
The well known Makeham’s Formula, see for instance (Broverman, 2010, p. 228) is a different look
of Formula (10.2), which has an interesting view.
76
Let us now talk on Current Yield, in this case for bonds, Bond Current Yield. In general it is seen
as an investment’s annual income (interest or dividend).
Definition 10.13 (Current Yield) Bond Current Yield is simply the annual interest earned
(Coupon) divided by the bond current market value (or Purchase, Market price)
C
P
Example 10.2 (Yield calculation) On the 02/01/2021 Zoroastro PLC, Zaratustra’s parent com-
pany, issued a bond loan of 200, 000 bonds with the following considerations:
• No. of Repayments: 4;
An investor bought one bond at issuance with redemption in the last date (02/01/2026). Calculate:
1. Investor’s Yield;
(9.20) 10 +
0.8 0.8 0.8 0.8 0.8
0 1 2 3 4 5
77
2. Investment Accumulated Value, S = F V , calculation comes:
r ' 10.117%
S = 0.8 · s5 r + 10 ' 14.89
The above calculation has an interesting (alternatively) view: If the investor had put the
amount e9.20 in a savings account, under rate r = 10.117% he would get exactly the same
above amount:
S = 9.2(1 + r)5 ' e14.89 .
Example 10.3 (Calculation of Purchase Value) For Example 10.2, consider that an investor
intends to buy, on 02/07/2023, 200 of these bonds so that they give him an Yield Rate of 9%. His
redemption plan was set as follows:
Calculate how much is the investor willing to pay for each bond?
Solution:
We build the following table,
10(0.08)
2/1/2023 200
= 160.00
2/1/2024 200 160.00 25 25 (10) = 250.00
2/1/2025 175 140.00 75 75 (10) = 750.00
2/1/2026 100 80.00 100 100(10) = 1, 000.00
78
10.4 Exercises
1. True/False questions:
(a) A bond loan issued above the par, also subscribed completely, gets a higher money inflow
than the loan value.
T F
(b) Consider a bond loan issued above the par and that redemption is paid under par.
Investors yield rate is lower than the coupon rate (annual). T F
(c) A bond loan, fully subscribed, changes the loan amount if units are issued above the
par. That is, the new loan amount is equal to the global nominal value plus the issuance
premia received.
T F
(d) A Zero Coupon bond does not pay interest.
T F
(e) A Zero Coupon bond loan is always more beneficial to the issuer because it does not pay
interest.
T F
(f) A zero coupon bond is considered to be a non-risky debt instrument.
T F
(g) The Maturity Value of a bond is its Redemption Value.
T F
(h) Failure to redeem a bond loan in due time affects the financial credibility of its issuer.
T F
(i) “T-Bills”, short for U.S. Treasury Bills, are considered to be risk-free investments.
T F
(j) Consider a bond loan issued at the par where redemption is paid with a premium (no
other expenses considered). Investors yield rate is higher than the coupon rate.
T F
(a) Zeppelin Company issued bonds worth e450, 000. The nominal value of each unit is
e 5.00 and the bond is issued at a discount of e 0.05. Compute the number of bonds
issued.
1. 90, 000 × ; 2. 100, 000 × ; 3. 81, 818 × ; 4. Other × .
(b) We have more information on the Zeppelin Company bond issuance above. All units
were placed at issue. Total revenue is:
1. e450, 000 × ; 2. e495, 000 × ;
3. Redemption Value is e450, 000 × ; 4. e445, 500 × .
(c) In a bond issue, if the issue value is less than the par value then it is considered to exist:
79
1. A fraud × ; 2. A profit for the issuer company × ;
3. A profit for the investor × ; 4. Bond is issued at a discount × .
(d) The company where Zuzanna works issued a bond loan. Issue was done at the par value,
redemption also at the par (no other expenses are considered). A bond is sold in the
market at the nominal value at the middle of a coupon payment period. The yield rate
to the selling investor, who had bought at the issuance date, is (relative to the coupon
rate):
80
2. Is lower than the coupon rate × ;
3. Information is missing × ;
4. Is equal to the coupon rate × .
3. Mr. Zacharjasz’s sharehold company, Omega, issued a bond loan with the following terms:
k Ik Nk Rk πk Πk Pk Bk
0 200, 000
1
2 48, 000
3
4
5
6 45, 200
4. Mrs. Zytas corporation issued a bond loan under the following terms:
81
k Ik Nt Rk πk Πk Pk Bk
0 2, 000, 000.00
1
2
3
4
5
5. Zach PLC issued a bond loan with the following technical terms:
(a) Compute the loan value, the issue premium and the cash inflow.
(b) Fill up the Amortization Table:
k Ik Nk Rk πk Πk Pk Bk
0 1, 200, 000.00
1
2
3
4
5
6
(c) Ms. Zyta bought 50 bonds immediately after payment of the 3rd coupon and kept them
until maturity (last redemption). He got an (excellent) yield rate of 11%.
Write the equation that allows to calculate how much money this investor spent for
buying those 50 bonds.
6. The public company where Mr. Zygfryd works with issued a bond loan with the following
technical aspects:
82
• Nominal Value: e 10.00;
• No. of bonds issued: 50, 000;
• Issue at the par value;
• Loan term: 3 years;
• Semi-annual coupon rate: 2.5%;
• Interest is paid semi-annually, at the end of every semester;
• Mode of Redemption: Repayments semi-annually of equal number of bond units, starting
1.5 years after the issue date;
• Redemption premium: 1st and 2nd repayments: e 0.25 per unit; e 0.50 per unit after
that.
83
Chapter 11
Share Valuation
11.1 Definitions
Bonds is a popular investment tool for the public as whole, units represent small money amounts,
and money savers can buy at least small amounts. It is relatively a safe instrument, it has small
risk, it has a safe and regular income, the operation is under control of the financial authorities in
each country.
Shares is another popular financial tool for investors, although riskier, also issued in units of small
money amounts. Shares values are more volatile, they don’t have a regular and safe income.
Unlike Bonds, Shares are not loans. They hold a property, a part of a company’s property. They
are called Stocks in United Sates and Share in United Kingdom. Like Bonds they are traded in
financial markets.
Definition 11.1 (Shares/Stocks) A Share (UK) or Stock (US) are equity securities (ownership
claim of a firm). It entitles the shareholder to an equal share in the ownership of a firm.
An equity security represents ownership interest held by holders in an entity (a company, part-
nership or trust). Usually, each share entitled to the same amount of profits and entitles to vote
on matters of corporate governance. Common shares represent a residual claim on the assets of a
firm (assets that are left over after meeting all of the firm’s other financial obligations). The only
companies that are allowed to issue shares are the Public Companies, fully owned by anonymous
shareholders, public in general.
Share prices or values may vary according to different concepts. The Share has always a Nominal
Value. Since they are traded they hold also a Market Value.
Definition 11.2 (Nominal Value) The Nominal Value is a Face Value, determined at issuance,
and corresponds to a percentage on the firm’s capital.
Definition 11.3 (Market Value) The Market Value is the value that is traded in the Market.
The Market Value depends on speculation, on the cash flows investors expect to receive in the
future, granted with the acquisition of the share (with either the sale or potential dividends, or
both).
84
11.2 Valuation
Value of a share depend very much on expectations, on expectations on market behaviour, more
rationally on how much investors are expecting to receive in the future from acquiring the share.
Shares give investors the right to receive future firm’s dividends. Dividends are distributed com-
pany’s profits, each shareholder is entitled to its corresponding portion.
Rationally, valuation of a share depend on estimation of future cash flows (money inflows minus
outflows), sizes and timing or “on how much and when”. Once estimated that you have to calculate
the whole discounted value at an appropriate rate. The discounting rate should be related with the
share risk, not just an interest rate.
As you can see, it depends on subject evaluation, and future evolution of cash flows, time, discount
rate, risk. Everything should be “appropriate”, that is the word, but no one is certain about it.
Uncertainty and subjectivity is always present.
Potential sources of Cash flows are: Future dividends to shareholders and Market value trading,
selling.
There are various models to evaluate quantitatively a share. We present three ways, through two
different problems.
Problem 11.1 (Share value: One year transaction) Consider that an investor buys and sells
in one year. We estimate a Future Value. See next diagram in Figure 11.1. Investor buys for P0 ,
sells for P1 one year later and estimate his Dividend D1 before selling. One year future Value is
discounted for the buy date with rate r representing the cost of capital, an opportunity cost.
−P0 D1 + P1
0 1
The discount rate, viewed as a return rate, is decomposed into two parts: D1 /P0 , as the Dividend
Yield, plus (P1 − P0 )/P0 , as the Capital Gain.
Example 11.1 (Share value. One year transaction) Zapata PLC is going to pay a dividend
of e0.56 per share next year and it has been estimated that the share can be sold at e 45.5 at the
end of the year.
Consider the following questions:
85
1. If another investment of similar risk has an expected return of 6.80%, how much will you
expect to pay for this share?
2. At the end of the year, what will the expected Dividend Yield and the Capital Gain be?
Solution:
The rate 6.8% is interpreted as an opportunity cost, or the return rate of an alternative investment.
1.
P0 = (0.56 + 45.5)/1.068 ' 43.13
2.
Second problem, where a share is kept for many years, either finite or infinite.
Problem 11.2 (Share value: Share kept for many years) Let Dk , k = 1, 2, . . . be the Divi-
dend estimate in year k, r is the return rate that is assumed to be constant.
There are some easy methodologies for estimating future dividends, making some Assumptions,
often simple, such as:
Problem 11.3 (Share value: Share kept for many years plus) To the Problem 11.2 above
add the following assumptions. It is known that: 1. Current or coming Dividend is known, denoted
as D; 2. Annual Growth Rate estimate for the coming dividends, denoted as g; 3. Discount Rate
estimate is known, denoted as r; 4. Horizon is infinite. Then, for r > g,
D
P0 = .
r−g
86
Proof.
!
1 1+g (1 + g)2
P0 = D + + + ...
1 + r (1 + r)2 (1 + r)3
n
1 − 1+g
1+r D
= D lim = if r > g .
n→∞ r−g r−g
Example 11.2 (Discount Rate) Zapata, PLC share is expected to pay a dividend of e 2 per
share a year from now, and its dividends are expected to grow at 6% per year thereafter.
If its price is now e 20 per share, what must be the discount rate?
Solution: Let r be the discount rate, then
2
20 = ⇔ r = 0.16 → r = 16%
r − 0.06
Example 11.3 (Share Valuation) Zapata Co. is expecting earnings of e10 per share, an earning
retention rate of 75%, an expected rate of return on future investments of 18% per year, and a
discount rate of 15% per year.
Compute an estimate of Zapata Co. share price.
Solution: Let r = 0.15 and g = 0.75 × 0.18 be the discount and the growth rate, respectively. Future
dividend at year k, k = 1, 2, . . . , is denoted as Dk . We know that 75% of the earnings are not
distributed as dividends, so future investments are done over that capital retained. Then,
D1 = 10.00 × 0.25
Dt+1 = Dt (1 + 0.75(0.18)) , t = 1, 2, . . . .
Discounting, we get
1 (1 + g) (1 + g)2
P0 = 2.5 + 2.5 + 2.5 + ...
1+r (1 + r)2 (1 + r)3
∞
X 1.135k 2.5 2.5
= 2.5 k+1
= = = 166.6(6)
1.15 0.015 0.15 − 0.75(0.18)
k=0
11.3 Exercises
1. True/False questions:
87
(c) A share is a financial security issued by companies to borrow money.
T F
(d) Shares of a public company pay interest if they are lent at issuance.
T F
(e) A share grants its owner to benefit from future profits of the company the share corre-
sponds to.
T F
(f) A bond is a share of a loan.
T F
(g) The market value of a stock is assured by the corresponding face or issue value.
T F
(h) A share does not assure every year dividend payments to the company’s shareholders.
T F
(i) Bonds and Shares can be both investments available to the public, organizations or
individuals.
T F
2. Market expectations suggest that a certain share pays yearly dividends, at the end of each
financial year, indefinitely. An investor, that wants to buy in the market some of those shares,
is available to buy for a price that gives a yield rate of 5%.
Calculate that price (per unit), for each of the following cases:
(a) Price equals the present value of a perpetuity with level payment e 2.
(b) Price equals the present value of an increasing geometric perpetuity with first payment
e 2 and growth rate 2%.
88
Solutions to Exercises
Chapter 1
2. (a) 3; (b) 2.
3. –
4. e961.54.
Chapter 2
Chapter 3
Chapter 4
1. (a) T; (b) T; (c) T; (d) T; (e) T; (f) T; (g) F; (h) F; (i) T; (j) F; (k) T .
2. (a) 2; (b) 1; (c) 1; (d) 3; (e) 1; (f) 1; (g) 2; (h) 2; (i) 3; (j) 1; (k) 3; (l) 3; (m) 2; (n) 4 .
89
Chapter 5
Chapter 6
1. (a) F; (b) T; (c) T; (d) T; (e) F; (f) F; (g) F; (h) T; (i) T; (j) F; (k) T; (l) F; (m) T; (n) T;
(o) F; (p) T; (q) T; (r) T; (s) T; (t) F; (u) T; (v) T; (w) T.
2. (a) 2; (b) 2; (c) 4 (e2 206, 6(6)); (d) 1; (e) 2; (f) 3; (g) 1; (h) 4; (i) 3; (j) 4 .
6. (a) e23, 446.10 , e17, 446.10; (b) e127, 012.10; (c) e23, 836.13; (d) e4, 471.29 .
Chapter 7
1. (a) T; (b) F .
3. (a) e 1 903, 86; (b) e 1 924, 64; (c) e 1 803, 96; (d) e 1 322, 26 .
Chapter 8
4. (a) P ∼ 1, 288.602, 803; (b) B1 ∼ 199, 711.3972 , B2 ∼ 199, 421.3514; (c) B12 ∼ 196, 439.95;
(d) Pk∗ ∼ 1, 360.807, 339; B299
∗ ∼ 1, 353.363, 838 .
7. (a) e797.14; (b) e667.94; (c) · · · ; (d) B3 = 22, 311.80 ; (e) B45 = 1, 954.74 .
90
Chapter 9
7. (a) e80, 000; e16, 302.41; (b) e4, 315.12; (c) e46, 633.83 .
Chapter 10
1. (a) T; (b) T; (c) F; (d) T; (e) F; (f) F, (g) T; (h) T; (i) T; (j) T .
2. (a) 1; (b) 4; (c) 4; (d) 3; (e) 1, (f) 3; (g) 2; (h) 3; (i) 3; (j) 2 .
3. P1 = 6, 000, P2 = 48, 000, P3 = 46, 800, P4 = 47, 600, P5 = 46, 400, P6 = 45, 200; B6 = 0.
5. (b) P1 = 36, 000, P2 = 444, 000, P3 = 24, 000, P4 = 436, 000, P5 = 12, 000, P6 = 424, 000;
B6 = 0.
6. (a) P1 = 12, 500, P2 = 12, 500, P3 = 140, 625, 000, P4 = 137, 500, P5 = 137, 500, P6 =
134, 375; B6 = 0.
(b) IRR ∼ 3.385%.
Chapter 11
91
Bibliography
Barroso, M., Couto, E., and Crespo, N. (2009). Cálculo e Instrumentos Financeiros–Da prática
para a teoria. Escolar Editora, 2nd ed., Lisboa.
Chan, W.-S. and Tse, Y.-K. (2017). Financial Mathematics for Actuaries. World Scientific Pub-
lishing Company, 2nd ed..
Guthrie, G. C. and Lemon, L. D. (2014). Mathematics of Interest Rates and Finance. Pearson
Higher Ed.
92